Google Groups no longer supports new Usenet posts or subscriptions. Historical content remains viewable.
Dismiss

What foreign languages did Shakespeare know, if any?

986 views
Skip to first unread message

M.R.Kohanna

unread,
Aug 9, 1999, 3:00:00 AM8/9/99
to
What foreign languages did Shakespeare know, if any?

I'm sure he knew some Latin, but was he fluent in French or any other language?
Thanks for any help.

++++++++++++++++++++++++++
M.R. Kohanna
3876 NORTHSIDE DR., APT. #8-A
Macon, GA 31210

John W. Kennedy

unread,
Aug 9, 1999, 3:00:00 AM8/9/99
to
"M.R.Kohanna" wrote:
>
> What foreign languages did Shakespeare know, if any?
>
> I'm sure he knew some Latin, but was he fluent in French or any other language?

Judging from "Henry V", he had some command of French. It seems safe to
say that he did _not_ know Spanish, Italian, Greek, Irish, Scots Gaelic,
Welsh, Cornish, or Breton. There's no evidence at all for other
languages -- Dutch is about the only other language an Englishman would
be likely to know, but there's no reason to think he knew it.

--
-John W. Kennedy
-rri...@ibm.net
Compact is becoming contract
Man only earns and pays. -- Charles Williams

Dave Kathman

unread,
Aug 9, 1999, 3:00:00 AM8/9/99
to
M.R.Kohanna wrote:
>
> What foreign languages did Shakespeare know, if any?
>
> I'm sure he knew some Latin, but was he fluent in French or any other language?
> Thanks for any help.

I don't know how fluent he was, but he certainly knew some French, as
evidenced by Henry V and a few other plays. Shakespeare lived with a
French family for a while, and a significant source for his
early-to-middle
plays and poems (written 1593-8) is John Eliot's French phrase book
*Orthoepia Gallica*, published by Shakespeare's fellow Stratfordian
Richard Field in 1593. Eliot was also from Warwickshire, so the three
men could have even known each other before 1593.

Dave Kathman
dj...@ix.netcom.com

Peter Farey

unread,
Aug 10, 1999, 3:00:00 AM8/10/99
to


The plot of *Othello* comes from *Disdemona and the Moor*, one
of the stories in Cinthio's *Hecatommithi* (1565). The first
extant English translation was in 1753, but there was certainly
a French translation (by Chappuys) available when *Othello* was
wrtten.

In his Arden edition of *Othello*, E.A.J. Honigmann provides
(Appendix 3, pp.368-387) quite convincing evidence that Shake-
peare was familiar with it not only in French but also in the
original Italian.

While he acknowledges that a lost translation in English
cannot be ruled out, he says that the cumulative weight of
the verbal and other debts to the non-English versions,
particularly the Italian, is "quite considerable".

Peter F.
pet...@rey.prestel.co.uk
http://www2.prestel.co.uk/rey/index.htm

Stephanie Caruana

unread,
Aug 14, 1999, 3:00:00 AM8/14/99
to
The answer to that depends on who you think Shakespeare was.

If you think he was the man from Stratford, Stratfordians generally allow
him at the least a slight knowledge of French, just enough for the very
rudimentary French scenes in Henry V. (Like, he only happened to know those
particular French words, used them up in a hurry, then had to fall back on
English). ... :-) (ROFL)

Over the years, some Shakespearean commentators have speculated that he knew
other languages as well. Someone even wrote a multi-volume series in the
mid-nineteenth century, called "Shakespeare's Library," listing some of the
books he would have needed in order to write what he wrote. Geoffrey
Bullough's multi-volume series on the narrative and dramatic sources of
Shakespeare includes hundreds of sources in a number of languages, many of
which were not available in translation when the "Shakespeare plays" were
appearing, and many of which are uncomfortably connected to Oxford. How
Shaksper of Stratford could have encountered these hundreds of sources has
been the subject of much uncomfortable writhing among the Stratfordian camp
for a couple of centuries, because the question admits of no believable
answer, except for the most unscientific one of the "bolt from the Blue"
hitting Stratford like the tornado that smashed Salt Lake City last week.
Only this one zapped William Shaksper and transformed him into that most
unlikely creature, a "natural man" who could know all there was to know,
without actually having to learn it. (Sigh....if only. Maybe that's the
appeal of the Stratfordian concept of Shaksper...if we could only figure out
what "smart pills" this guy took, maybe we could do it too????)

If you think Shakespeare might have been Oxford, the picture is different.
B.M. Ward quotes Oxford's daily routine as a minor in Cecil House, from the
State Papers of Elizabeth I's reign. They include 1 hour of French,
followed by 1 hour of Latin, in the morning; and in the afternoon, 1 more
hour of Latin, followed by 1 more hour of French (as well as other subjects
to round out the days' lessons). Then there is the letter in French, dated
August 23, 1563, written by Oxford to his guardian, Cecil. The penmanship
is quite attractive, and legible,and the French is quite acceptable. [If
anyone is interested, I will post the French text here.] One may assume
without too much of a stretch that this is an "exercise with his pen," one
of the daily lessons included in Oxford's daily school routine.

In addition, Oxford was tutored in Latin by Lawrence Nowell, Dean of
Lichfield, an outstanding Latinist, collector of antiquities and Old English
manuscripts, owner of the original and unique manuscript of Beowulf, etc.,
etc., and brother of Alexander Nowell, Dean of St. Paul's. In addition,
Oxford was a student at both Oxford and Cambridge, where study in Latin,
Greek and Hebrew were required.

In addition, Oxford studied law at Gray's Inn, and became a lawyer, and a
judge (tryer of petitions), which explains why he includes so many legal
terms and concepts in his plays, and why so many distinguished lawyers and
jurists, such as U.S. Supreme Court Justice Stevens, have been able to
understand him so well.

In addition, Oxford spent at least a year in Italy, and with his natural
aptitude for language, it's difficult to avoid imagining that at least a
phrase or two rubbed off on him. I mean, at least, "Pardon me, if you
please, which way to the men's room?"

In addition, there are the receipts for Plutarch's Lives, in Latin, and
numerous other books which were purchased for Oxford when he was still in
his teens. And there's that annotated copy of Oxford's Bible, elegantly
bound with his coat of arms stamped in gold on the cover, that lurked
unnoticed in the Folger Library and then was spotted in a display case by an
Oxfordian, Dr. Paul Nelson, of Ohio. And then of course he did write a
couple of literary prefaces and poems and such in Latin, as well as in
English.

But if you still insist on William of Stratford as Mr. Miracle Man, what
more can I say?


Stephanie Caruana
Spear Shaker Review - On-line Quarterly Oxfordian Magazine
http://www.spear-shaker-review.com

M.R.Kohanna wrote in message
<19990809143318...@ng-cn1.aol.com>...


>What foreign languages did Shakespeare know, if any?
>
>I'm sure he knew some Latin, but was he fluent in French or any other
language?
>Thanks for any help.
>

Greg Reynolds

unread,
Aug 15, 1999, 3:00:00 AM8/15/99
to
M.R. Kohanna wrote:

The answer does NOT depend on opinions at all. The question is "What foreign languages did Shakespeare know, if any?"  There is only one true answer and it only depends on what languages he knew, not on who you think Shakespeare was.

Okay, let's attend to your answer. We want to collect information to reach the truth. Because its the Sabbath, I won't work too hard here, I'll allow conjecture. But not mistruths. (I'm sure you'll be talking about Ben Jonson's quotes on Shakespeare's knowledge of languages--its contemporary, reliable, and central to the matter.)

 If you think he was the man from Stratford, Stratfordians generally allow
him at the least a slight knowledge of French, just enough for the very
rudimentary French scenes in Henry V. (Like, he only happened to know those
particular French words,  used them up in a hurry, then had to fall back on
English). ...  :-) (ROFL)

not a pretty sight

Over the years, some Shakespearean commentators have speculated that he knew
other languages as well. Someone even wrote a multi-volume series in the
mid-nineteenth century, called "Shakespeare's Library," listing some of the
books he would have needed in order to write what he wrote.

Come clean. Who is your mystery deep throat?
Unless Sh was plagiarizing, he would not need the books, just mention of the appropriate contents he was recreating. Please supply the name of a book he would need but couldn't easily get.

Less anecdotes, more detail.

Geoffrey
Bullough's multi-volume series on the narrative and dramatic sources of
Shakespeare  includes hundreds of sources in a number of languages, many of
which were not available in translation when the "Shakespeare plays" were
appearing, and many of which are uncomfortably connected to Oxford.

Are you hypnotized?
Provide any facts that Sh needed a book he couldn't get.
What does "uncomfortably connected to Oxford" mean (except for that one italian boy)?

 How
Shaksper of Stratford could have encountered these hundreds of sources has
been the subject of much uncomfortable writhing among the Stratfordian camp
for a couple of centuries,

1799? writhing? The oxford hoax wasn't invented yet.

You're using 'uncomfortable' a lot.

because the question admits of no believable
answer, except for the most unscientific one of the "bolt from the Blue"
hitting Stratford like the tornado that smashed Salt Lake City last week.

Here you use an actual recorded event, a tornado, as an example of a supposed impossible event, a sudden enlightening. There are similes more apt. You're a writer?

We're well into your response to M.R. Kohanna's simple question and you've avoided the historical record of Jonson speaking on Shakespeare. Is it "uncomfortably connected" to something?

Only this one zapped William Shaksper and transformed him into that most
unlikely creature, a "natural man" who could know all there was to know,
without actually having to learn it.  (Sigh....if only.  Maybe that's the
appeal of the Stratfordian concept of Shaksper...if we could only figure out
what "smart pills" this guy took, maybe we could do it too????)

You lost me on this one. Smart pill? What are you peddling, Stephanie?
Mention Jonson or I will.

If you think Shakespeare might have been Oxford, the picture is different.

I'll say. And boars fly.

B.M. Ward quotes Oxford's daily routine as a minor in Cecil House, from the
State Papers of Elizabeth I's reign.  They include 1 hour of French,
followed by 1 hour of Latin, in the morning; and in the afternoon, 1 more
hour of Latin, followed by 1 more hour of French (as well as other subjects
to round out the days' lessons). Then there is the letter in French, dated
August 23, 1563, written by Oxford to his guardian, Cecil.  The penmanship
is quite attractive, and legible,and the French is quite acceptable. [If
anyone is interested, I will post the French text here.]

For free or do we have to pay?
Post its link, and I'll get you a translation from the era. (The question was not about Oxford's knowledge of languages, though.)

One may assume
without too much of a stretch that this is an "exercise with his pen,"  one
of the daily lessons included in Oxford's daily school routine.

Let's just see it. We can leave the assuming to the experts.

In addition, Oxford was tutored in Latin by Lawrence Nowell, Dean of
Lichfield, an outstanding Latinist, collector of antiquities and Old English
manuscripts, owner of the original and unique manuscript of Beowulf, etc.,
etc., and brother of Alexander Nowell, Dean of St. Paul's.  In addition,
Oxford was a student at both Oxford and Cambridge, where study in Latin,
Greek and Hebrew were required.

Did he write in these languages?

In addition, Oxford studied law at Gray's Inn, and became a lawyer, and a
judge (tryer of petitions),  which explains why he includes so many legal
terms and concepts in his plays, and why so many distinguished lawyers and
jurists, such as U.S. Supreme Court Justice Stevens, have been able to
understand him so well.

Stevens ruled against the claim that Oxford wrote Shakespeare. What are you dreaming? (Quit pretending Oxford won on appeal. The verdict was 3-0.)

Where was all this clever legalese in the plays? Portia's clarification?

In addition, Oxford spent at least a year in Italy, and with his natural
aptitude for language, it's difficult to avoid imagining that at least a
phrase or two rubbed off on him.  I mean, at least, "Pardon me, if you
please, which way to the men's room?"

I don't think he needed directions there.
Did he write in Italian? Its important to know and I'll explain.

In addition, there are the receipts for Plutarch's Lives, in Latin, and
numerous other books which were purchased for Oxford when he was still in
his teens.  And there's that annotated copy of Oxford's Bible, elegantly
bound with his coat of arms stamped in gold on the cover, that lurked
unnoticed in the Folger Library and then was spotted in a display case by an
Oxfordian, Dr. Paul Nelson, of Ohio.  And then of course he did write a
couple of literary prefaces and poems and such in Latin, as well as in
English.

And all those mining proposals.

But if you still insist on William of Stratford as Mr. Miracle Man, what
more  can I say?

Just say you have no idea what languages Shakespeare knew. You don't.

Here's Ben Jonson--he does know:

from TO THE MEMORY OF MY BELOVED THE AUTHOR, MR. WILLIAM SHAKESPEARE:

25   That I not mix thee so, my brain excuses,
26   I mean with great, but disproportion'd Muses,
27   For if I thought my judgment were of years,
28   I should commit thee surely with thy peers,
29   And tell how far thou didst our Lyly outshine,
30   Or sporting Kyd, or Marlowe's mighty line.
31   And though thou hadst small Latin and less Greek,
32   From thence to honour thee, I would not seek
33   For names; but call forth thund'ring {AE}schylus,
34   Euripides and Sophocles to us;

So Shakespeare was familiar with Latin and Greek (but probably not as fluent as Jonson himself). Complete text at http://www.library.utoronto.ca/utel/rp/poems/jonson15.html

If Jonson is aware that Oxford is Shakespeare, he doesn't mention the Italian, French, or Hebrew. So I doubt that Oxford was the writer of Shakespeare. Because Jonson certainly knew Shakespeare.

Stephanie, are you uncomfortable with Jonson? You seem to ignore his up close and personal account of Shakespeare's knowledge of languages. It doesn't agree with yours, but his is the historical account and yours is a business (http://www.spear-shaker-review.com)

Greg Reynolds
 
 

Nigel Davies

unread,
Aug 15, 1999, 3:00:00 AM8/15/99
to
Stephanie Caruana wrote:

> The answer to that depends on who you think Shakespeare was.
>
> If you think he was the man from Stratford, Stratfordians generally allow
> him at the least a slight knowledge of French, just enough for the very
> rudimentary French scenes in Henry V. (Like, he only happened to know those
> particular French words, used them up in a hurry, then had to fall back on
> English). ... :-) (ROFL)
>
> Over the years, some Shakespearean commentators have speculated that he knew
> other languages as well. Someone even wrote a multi-volume series in the
> mid-nineteenth century, called "Shakespeare's Library," listing some of the
> books he would have needed in order to write what he wrote. Geoffrey

> Bullough's multi-volume series on the narrative and dramatic sources of
> Shakespeare includes hundreds of sources in a number of languages, many of
> which were not available in translation when the "Shakespeare plays" were
> appearing, and many of which are uncomfortably connected to Oxford...

Yes, indeed. Shakespeare never engaged in any human contact, did he? He just sat
in a room all alone relying entirely on his own abilities in every field.
Everything he wrote had to be sourced from written literature. He had to have
been fully educated in any language he used, no matter how small the reference.
He had to have been a master musician for all the musical interludes he included
in his plays. He had to have no contact with anyone else who might be able to
add value to his plays in any particular field. I mean, Shakespeare actually
collaborating with anyone else in compiling a play? Surely not! It never
happened, did it?

Of course, if you had actually read Shakespeare's plays you would know that the
proficiency he held in these languages you cite is vastly inferior to the dizzy
heights you elevate them to. And, if you'd researched Shakespeare's life you'd
know of proficient multi-lingual friends like John Florio whom he had available
to contribute any linguistic exercises he desired to spice up his plays. And, if
you had actually read the sonnets you would know the play on Latin word origins
and meanings he predominantly uses that betray his Latin Grammar education but
also note the conspicuous absence of play on other languages such as Greek that
betray his lack of competence in those areas. And, you would check very
carefully on your sources that you claim did not have English translations and
could not possibly have had anyone else in cosmopolitan London available whom
Shakespeare knew able to translate for him without doing so for commercial
profit. I vividly recall Volker's triumphant claim that one of Shakespeare's
sources, Belleforest, was only available to French readers, in blissful
ignorance that Belleforest's "Histoires Tragiques" was translated into English
by Sir Geoffrey Fenton under the title "Certaine Tragicall Discourses written
oute of Frenche and Latin" in 1567, when Shakespeare was just 3 years old.

Given your glaringly inept assignment of Paul McCartney's "Golden Slumbers" to
John Lennon you probably think he also had to have a degree in French to write
"Michelle" despite its rudimentary use of the language and even then him
requiring a French girlfriend at the time to write the French verses.
______________________________________________________________________
nda...@emirates.net.ae

Stephanie Caruana

unread,
Aug 15, 1999, 3:00:00 AM8/15/99
to
To Greg Reynolds:

I am posting a copy of the French letter Oxford wrote to his guardian, Lord
Burghley, when he was 13 years old, as you requested. I have left out the
accent marks and cedillas of the original since my keyboard is not set up
for French. Since you apparently don't know French, it won't matter to you
anyway; and if you did know French, you would be able to figure them out for
yourself.

August 23, 1563

Monsieur tres honorable,

Monsieur, j'ai recu vos lettres plaines d'humanite et courtoisie, et fort
resemblantes a votre grand amour et singulier affection envers moi, comme
vrais enfants devement procrees d'une telle mere pour laquelle je me trouve
de jour en jour plus tenu a v.h.vos bons admonestements pour l'observation
du bon ordre selon vos appointements. Je me delibere (Dieu aidant) de garder
en toute diligence comme chose que je cognois et considere tendre
especialment a mon propre bien et profit, usant en cela l'advis et authorite
de ceux qui sont aupres de moi, la discretion desquels j'estime si grande
(s'il me convient parler quelquechose a leur avantage) qui non seulement ils
se porteront selon qu'un tel temps le requiert, ains que plus est feront
tant que je me gouverne selon que vous avez ordonne et commande. Quant a
l'ordre do mon etude pour ce qu'il requiert on long discours a l'expliquer
par le menu, et le temps est court a cette heure, je vous prie
affectueusement m'en excuser pour le present, vous assurant que par le
premier passant je le vous ferai savoir bien au long. Cependant je prie a
Dieu vous donner sante.

Edward Oxinford

Since you profess to be so concerned with "evidence," perhaps you will take
the time to compare the text of the above letter with the scene in Henry V
(Act 3, scene 4) where the French princess, Katherine, is getting an English
lesson from her maid, Alice. The subject matter of the two brief examples
is different, and Oxford addresses his guardian in a more formal style,
while Katherine and her maid use the familiar form of pronouns, etc. Even
so, a number of words/phrases occur in both selections:

j'ai (I have)
et (and)
votre (your)
Dieu (God)
temps (time)
vous (you)
je (I)
le (the)
vous avez (you have)
que (what)


From Act 5, scene 2, where Henry woos Katherine in broken English:
plaines d' (full of)


From Act 4, Scene 4, where Pistol is bullying a French solider, can be
gleaned a few more words used in common:
a cette heur (at this time)
bon, bonne (good)
commande, commande' (order, ordered)
monsieur (sir)
grande (great

It appears to me that this constitutes evidence for the likelihood that
Oxford knew enough French to have written these scenes.

Now let me ask you the same thing you have asked of me: What EVIDENCE do
you have that William Shaxper of STRATFORD knew even enough French to be
able to write those scenes in Henry V? Try to wrap your brain around the
idea that citing the play itself is not proof that Shaxper knew French,
since the identity of the author is the question at issue.

Stephanie Caruana wrote:
Then there is the letter in French, dated
August 23, 1563, written by Oxford to his guardian, Cecil. The
penmanship
is quite attractive, and legible,and the French is quite acceptable. [If
anyone is interested, I will post the French text here.]

Greg Reynolds wrote:
For free or do we have to pay?
Post its link, and I'll get you a translation from the era. (The question
was not about Oxford's knowledge of languages, though.)
One may assume
without too much of a stretch that this is an "exercise with his pen,"
one

of the daily lessons included in Oxford's daily school routine. -sc
Let's just see it. We can leave the assuming to the experts. --GR


If you have a hard time getting "a translation from the era," whatever
that means, I will be happy to provide one.

As for the other "points" you "made" in your letter, to reply adequately
would take about as many volumes as Bullough's work and I don't time for
that.
I doubt that anything would penetrate into your thought processes, anyway.

Symposium1

unread,
Aug 15, 1999, 3:00:00 AM8/15/99
to
Oh, Stephanie, I've actually been hoping you would shed some light on this
argument and show up these people who have been picking on you. But this is so
ridiculous an argument, even I had to laugh at it.

The idea that the common use of "I", "you", and "the" in French shows any kind
of linkage from Oxford to Shakespeare is pretty ludicrous. You and I both used
all three words, and we are decidedly not the same person! They happen to be
among the most common words in any language.

Now, if Oxford had used a specific error or idiosyncrasy that Shakespeare had
repeated and NO ONE else had repeated the same flaw, that might be something.
Even still, the flaw might be more common than we could ascertain, and the
remaining records simply lost due to time and the impermanence of paper. And
it still wouldn't rule out the possibility that Shakespeare and Oxford were in
cahoots in some fashion other than Shakespeare being a simple straw man.

In article <7p6sbj$lar$1...@nntp6.atl.mindspring.net>, "Stephanie Caruana"
<spear-...@mindspring.com> writes:

>Even so, a number of words/phrases occur in both selections:
>
>j'ai (I have)
>et (and)
>votre (your)
>Dieu (God)
>temps (time)
>vous (you)
>je (I)
>le (the)
>vous avez (you have)
>que (what)

Qu'est-ce que vous avez ici? Mon Dieu! Je pense, donc je suis. Et vous?
Etes-vous?

--Ann


Greg Reynolds

unread,
Aug 15, 1999, 3:00:00 AM8/15/99
to

Stephanie Caruana wrote:

> To Greg Reynolds:
>
> I am posting a copy of the French letter Oxford wrote to his guardian, Lord
> Burghley, when he was 13 years old, as you requested. I have left out the
> accent marks and cedillas of the original since my keyboard is not set up
> for French. Since you apparently don't know French, it won't matter to you
> anyway; and if you did know French, you would be able to figure them out for
> yourself.

Thank you. I will be relying on an expert for a translation.

> August 23, 1563
>
> Monsieur tres honorable,
>
> Monsieur, j'ai recu vos lettres plaines d'humanite et courtoisie, et fort
> resemblantes a votre grand amour et singulier affection envers moi, comme
> vrais enfants devement procrees d'une telle mere pour laquelle je me trouve
> de jour en jour plus tenu a v.h.vos bons admonestements pour l'observation
> du bon ordre selon vos appointements. Je me delibere (Dieu aidant) de garder
> en toute diligence comme chose que je cognois et considere tendre
> especialment a mon propre bien et profit, usant en cela l'advis et authorite
> de ceux qui sont aupres de moi, la discretion desquels j'estime si grande
> (s'il me convient parler quelquechose a leur avantage) qui non seulement ils
> se porteront selon qu'un tel temps le requiert, ains que plus est feront
> tant que je me gouverne selon que vous avez ordonne et commande. Quant a
> l'ordre do mon etude pour ce qu'il requiert on long discours a l'expliquer
> par le menu, et le temps est court a cette heure, je vous prie
> affectueusement m'en excuser pour le present, vous assurant que par le
> premier passant je le vous ferai savoir bien au long. Cependant je prie a
> Dieu vous donner sante.
>
> Edward Oxinford
>
> Since you profess to be so concerned with "evidence," perhaps you will take
> the time to compare the text of the above letter with the scene in Henry V
> (Act 3, scene 4) where the French princess, Katherine, is getting an English
> lesson from her maid, Alice. The subject matter of the two brief examples
> is different, and Oxford addresses his guardian in a more formal style,

> while Katherine and her maid use the familiar form of pronouns, etc. Even


> so, a number of words/phrases occur in both selections:
>
> j'ai (I have)
> et (and)
> votre (your)
> Dieu (God)
> temps (time)
> vous (you)
> je (I)
> le (the)
> vous avez (you have)
> que (what)

Where DON'T these words show up? They are as basic as they come.

> From Act 5, scene 2, where Henry woos Katherine in broken English:
> plaines d' (full of)
>
> From Act 4, Scene 4, where Pistol is bullying a French solider, can be
> gleaned a few more words used in common:
> a cette heur (at this time)
> bon, bonne (good)
> commande, commande' (order, ordered)
> monsieur (sir)
> grande (great

Startling!

> It appears to me that this constitutes evidence for the likelihood that
> Oxford knew enough French to have written these scenes.
>
> Now let me ask you the same thing you have asked of me: What EVIDENCE do
> you have that William Shaxper of STRATFORD knew even enough French to be
> able to write those scenes in Henry V?

Oh, he was resourceful, and could get it done within seconds--especially if the
players knew French. This is not very hard to imagine. Any french speaker could
do it. That the audience understood the French indicates this was no big deal.

> Try to wrap your brain around the
> idea that citing the play itself is not proof that Shaxper knew French,
> since the identity of the author is the question at issue.

I didn't cite the play. I'm only staying on the original question. This
sideline, that Oxford knew French, is beside the point--but kinda fun.

> If you have a hard time getting "a translation from the era," whatever
> that means, I will be happy to provide one.

Unless my expert is on holiday, I may have a translation very soon. But,
please, get yours ready and we'll have a showdown. They may both be perfect and
identical--or we may learn a lot. Don't post til we are both prepared, please.
I'll give you my progress.

'From the era' means just that--my french expert is a historian as well and
will provide the proper usage for 1563. Any other approach is pointless.

> As for the other "points" you "made" in your letter, to reply adequately
> would take about as many volumes as Bullough's work and I don't time for
> that.

No comment.

> I doubt that anything would penetrate into your thought processes, anyway.
>
> Stephanie Caruana

You mean I was born and will die with this one mindset?
No, ma'am, I learn all the time. Has to make sense, though.

Looking forward to the 1563 French showdown. Tell me when you are ready.

Finding time for Stephanie,
Greg Reynolds


MILKY WAY

unread,
Aug 15, 1999, 3:00:00 AM8/15/99
to
I have a feeling he knew all the Romance languages. Which are French,
Spanish, and Italian. These are all based in Latin. The black legend
probable insured that next to none of Shakespeare's work survived in
Spanish. Although Cervantes has a very close birthday with William, and
they both use the same lines in their work. The Greatest book
(originally a series of stories) this Earth has ever seen is Don
Quixote. Without this book the USA could not have been formed the way it
is today.
Indo-European;
Indo-Iranian
Anatolian
Armenian
Hellenic
Tocharian
Celtic
Albanian, Baltic, Slavic,
Germanic (West Germanic, Old English, Middle English, English)
Italic: (Faliscan, Umbrian, Oscan)
Latin
Spanish
Ladino
Portuguese
Catalan
French
Provencal
Italian
Sardinian
Rhaeto-Romanic
Rumanian

On Second thought Cervantes went through the school of hard-knox, and
probable wasn't William Shakespeare. Not that Shakespeare didn't have
tragedy in his upper-class life.

"Miguel De Cervantes Saavedra, the son of a poor Spanish doctor, was
born in 1547 and took service in Italy when he was twenty-one. As a
regular soldier he took part in the naval battle of Lepanto and other
engagements until he was captured by pirates while returning to Spain in
1575 and taken to be the slave of a renegade Greek in Algiers; he
attempted unsuccessfully to escape three times and was finally ransomed
in 1580. For the rest of his life he was preoccupied with the
difficulties of making a living and spent several periods in prison. He
had already written some plays and a pastoral novel, La Galatea, when in
1592 he offered to write six plays at fifty ducats apiece, each to be
one of the best produced in Spain. He had no success until 1604, when
the publication of the first part of Don Quixote brought him immediate
popularity. A collection of short stories, The Exemplary Novels, was
published in 1613, and in 1614 appeared the promised continuation of Don
Quixote. Cervantes died in 1615." J.M. COHEN.


David L. Webb

unread,
Aug 16, 1999, 3:00:00 AM8/16/99
to
In article <7p6sbj$lar$1...@nntp6.atl.mindspring.net>, "Stephanie Caruana"
<spear-...@mindspring.com> wrote:

> To Greg Reynolds:
>
> I am posting a copy of the French letter Oxford wrote to his guardian, Lord
> Burghley, when he was 13 years old, as you requested. I have left out the
> accent marks and cedillas of the original since my keyboard is not set up
> for French. Since you apparently don't know French, it won't matter to you
> anyway; and if you did know French, you would be able to figure them out for
> yourself.

> August 23, 1563

> From Act 5, scene 2, where Henry woos Katherine in broken English:
> plaines d' (full of)

> From Act 4, Scene 4, where Pistol is bullying a French solider, can be
> gleaned a few more words used in common:
> a cette heur (at this time)
> bon, bonne (good)
> commande, commande' (order, ordered)
> monsieur (sir)
> grande (great

[...]

Consider the following selection (as you did, I will omit accents, as
default keyboard conventions seem to differ significantly).

"Chere et excellente amie. Votre lettre du 13 m'a cause une grande
joie. Vous m'aimez donc toujours, ma poetique Julie. L'absence dont vous
dites tant de mal, n'a donc pas eu son influence habituelle sur vous. Vous
vous plaignez de l'absence -- que devrai-je dire moi si j'osais me
plaindre, privee de tous ceux qui me sont chers? Ah! Si nous n'avions pas
la religion pour nous consoler, la vie serait bien triste. Pourquoi me
supposez vous un regard severe quand vous me parlez de votre affection
pour le jeune homme? Sous ce rapport je ne suis rigide que pour moi. Je
comprends ces sentiments chez les autres et si je ne puis approuver ne les
ayant jamais ressentis, je ne les condamne pas. Il me parait seulement que
l'amour chretien, l'amour du prochain, l'amour pour ses ennemis est plus
meritoire, plus doux, et plus beau, que ne le sont les sentiments que
peuvent inspirer les beaux yeux d'un jeune homme a une jeune fille
poetique et aimante que vous.
La nouvelle de la mort du comte Bezukhov nous est parvenue avant votre
lettre, et mon pere en a ete tres affecte. Il dit que c'etait
l'avant-dernier representant du grand siecle, et que present c'est son
tour; mais qu'il fera son possible pour que son tour vienne le plus tard
possible. Que Dieu nous garde de ce terrible malheur!"

In this brief selection, one finds the following words from the letter of
Oxford you quoted above:

et
votre
lettre
une
grande
vous
me
tant
de
que
je
moi
si
qui
sont
pour
la
un
affection
ce
seulement
amour
du
plus
mon
il
present
le
Dieu
garde(r)
comme


If I continued with the rest of the letter (but I won't bother to type any
more of it), there would be many more words in common with Oxford's
letter. Yet this extract was chosen by opening the text of Tolstoy's _War
and Peace_, at random, to a French passage. Should we imitate your
reasoning and infer that Oxford wrote _War and Peace_? That the French
prose of a Russian novelist writing some three centuries after Oxford
should exhibit so many words in common with the letter of Oxford you
quoted ought to disabuse you decisively of the fantastic notion that the
use of a few very commonplace French words in both Oxford and Shakespeare
has any bearing whatever on authorship. I suspect that one would find the
same correspondences in almost any modern French prose one examined
(except, of course, a text like Perec's _La Disparition_ or some similar
Oulipian work).

I really don't understand your point here. Why do you emphasize the
French words used "in common" by Oxford and Shakespeare as evidence?
Evidence of what? Certainly not evidence of common authorship of
Oxford's letter and the _Henry V_ French scenes.

Evidence that Oxford knew enough French to have written the works
attributed to Shakespeare? That's hardly news. Plenty of people fall
into that category.

> I doubt that anything would penetrate into your thought processes, anyway.

Actually, from Greg's past posts it's clear that quite a bit has
"penetrated into his thought processes"; some of his points are well
taken. I can assure you that quite a bit has penetrated into my thought
processes over the years as well, and I'm just as skeptical as Greg is
(perhaps more so, as I'm not even sure what your point is). I can
scarcely believe that we were meant to take your argument by enumeration
of common French words seriously. Please reassure me that you were just
trying to outdo Art Neuendorffer in entertainment value, or perhaps to
induce readers to subscribe to "Spear Shaker Review" as a source of comic
amusement.

Speaking of evidence, I'm still waiting for *any* hard evidence that
Ward had any idea (other than that prompted by wishful thinking) when the
dowager Countess of Oxford remarried. I'm also still awaiting a citation
proving that Delia Bacon, regarded even by nearly every Oxfordian I've
read as a proponent of a group theory led by Bacon and Raleigh, identified
Oxford as the leader of the Shakespeare authorship cabal. Finally, I'd be
interested in evidence that the Beatles' song "Golden Slumbers" (whose
authorship is not in doubt) "could have come from deVere's pen," as you
speculated a few weeks ago. I'll even make it worth your while: if you
provide hard evidence that any *one* of these claims is correct, I'll
subscribe to "Spear Shaker Review" for a year. Of course, if you cannot
do so, it would seem to be incumbent upon you to abandon these beliefs,
and in the interest of intellectual honesty, I would hope to see an
Editorial in "Spear Shaker Review" acknowledging that Oxford's mother's
"hasty" remarriage, a staple of the Oxfordian reading of _Hamlet_, seems
to be without foundation.

David Webb

Nigel Davies

unread,
Aug 16, 1999, 3:00:00 AM8/16/99
to

What evidence do you have that this letter was actually written by
Oxford? You have the temerity to question the authorship of the works
signed by William Shakespeare yet have Oxford holding the pen of this
letter. Are you absolutely sure that the young Earl didn't have this
written for him?

> Since you profess to be so concerned with "evidence," perhaps you will take
> the time to compare the text of the above letter with the scene in Henry V
> (Act 3, scene 4) where the French princess, Katherine, is getting an English
> lesson from her maid, Alice. The subject matter of the two brief examples
> is different, and Oxford addresses his guardian in a more formal style,
> while Katherine and her maid use the familiar form of pronouns, etc. Even
> so, a number of words/phrases occur in both selections:
>
> j'ai (I have)
> et (and)
> votre (your)
> Dieu (God)
> temps (time)
> vous (you)
> je (I)
> le (the)
> vous avez (you have)
> que (what)

Are you seriously suggesting the presence of these commonplace French
words is evidence for Oxford? "I", "and", "your", "you", "what"? You
must be more desperate than we thought. Using your logic Oxford also
wrote the French National Anthem, Camus's "Le Premier Homme" and
Michelin's "Paris Restaurant Guide".

> From Act 5, scene 2, where Henry woos Katherine in broken English:
> plaines d' (full of)
>
> From Act 4, Scene 4, where Pistol is bullying a French solider, can be
> gleaned a few more words used in common:
> a cette heur (at this time)
> bon, bonne (good)
> commande, commande' (order, ordered)
> monsieur (sir)
> grande (great
>

> It appears to me that this constitutes evidence for the likelihood that
> Oxford knew enough French to have written these scenes.

It appears to me you have sunk to a new low in Oxfordian desperation.
The presence of the word "Monsieur" in a letter and Henry V is striking,
is it? Can you show me any French correspondence to a male recipient
that doesn't contain the word "Monsieur"? What an extraordinary feeble
argument you have. No wonder all the other Oxfordians have gone quiet on
you.

> Now let me ask you the same thing you have asked of me: What EVIDENCE do
> you have that William Shaxper of STRATFORD knew even enough French to be

> able to write those scenes in Henry V? Try to wrap your brain around the


> idea that citing the play itself is not proof that Shaxper knew French,
> since the identity of the author is the question at issue.

The evidence is that any of a number of people could have helped him
with those lines in Henry V if needed. He lived with French people for a
time. It's an enormous stretch of your imagination to claim that an
author who lived with French people for part of his life didn't have the
means necessary to write a few lines in French.

> Stephanie Caruana wrote:
> Then there is the letter in French, dated
> August 23, 1563, written by Oxford to his guardian, Cecil. The
> penmanship
> is quite attractive, and legible,and the French is quite acceptable. [If
> anyone is interested, I will post the French text here.]

> Greg Reynolds wrote:
> For free or do we have to pay?
> Post its link, and I'll get you a translation from the era. (The question
> was not about Oxford's knowledge of languages, though.)
> One may assume
> without too much of a stretch that this is an "exercise with his pen,"
> one

> of the daily lessons included in Oxford's daily school routine. -sc
> Let's just see it. We can leave the assuming to the experts. --GR


>
> If you have a hard time getting "a translation from the era," whatever
> that means, I will be happy to provide one.
>

> As for the other "points" you "made" in your letter, to reply adequately
> would take about as many volumes as Bullough's work and I don't time for
> that.

> I doubt that anything would penetrate into your thought processes, anyway.
>

> Stephanie Caruana
> Spear Shaker Review - On-line Quarterly Oxfordian Magazine
> http://www.spear-shaker-review.com

______________________________________________________________________________
nda...@emirates.net.ae


Stephanie Caruana

unread,
Aug 16, 1999, 3:00:00 AM8/16/99
to
Anne,

You wrote:

>Oh, Stephanie, I've actually been hoping you would shed some light on this
>argument and show up these people who have been picking on you

Obviously, I wasn't trying to prove that this letter written by a
13-year-old boy was an early draft of Hamlet! Or that any of the specific
words were significant. I had about a 200-word sample, on another subject,
so of course the most common words would be the only matches. Please don't
get me wrong. I was really trying to see whether there was any level of
"evidence", no matter how trivial, that would satisfy some of these guys'
silly demands, and to ask for a little "evidence" from them in return, which
is never, ever, forthcoming.

We have "evidence, " in his own handwriting, signed, that at age 13, Oxford
knew French, enough to write the French scenes in Henry V.

The Stratfordians have 6 "signatures," all spelled differently, on legal
documents, and never a "Mais oui" among them. They have no "evidence" that
Shaxper even knew how to spell his own name, much less write a passage in
French, no matter how elementary.

When pushed, they
immediately run behind the oozing malt sack barricades and mumble that of
course, Florio was there to help out, and somebody must have known some
French around there. [Where?] Then there's the French Wigmaker landlord,
etc. (Maybe he had some French postcards? Oops, no post office, so that's
out....) That's their "evidence."

If you'll note, there are a few replies, with the obligatory abuse, floating
around to that effect right now, but I won't bother to cite them.

One of them that I just noticed is even "demanding" that I "prove Oxford
didn't have someone write the letter for him." Demanding?????

Come on, this is "literary discussion?"

So let me ask (again) any of these brave Strats to present any real evidence
they
may have hidden among the vast corridors of Shakesperoid "research" to
bring forth a few words of real "evidence" that William Shaxper of Stratford
ever wrote a word of French in his life. Maybe they can find some nice
hitherto unnoticed Collier forgery that will back them up? Oh, he's dead...
too bad, he has been so useful!

I'd say we're ahead so far, "on the evidence." :-)

I wish it were possible for Oxfordians to post on this newsgroup without the
Brown Shirt School of Literary Criticism feeling it their duty to try to
insult, hassle, abuse and harass us into non-participation.

It might then be possible for others to participate more freely in open
discussion as well without fear of being flamed for expressing an opinion of
their own.

But since the Brown Shirts are always on duty with their billy clubs, there
really isn't much opportunity for open exploration of ideas.

Maybe if we ignore them, they will eventually get bored and wander off to
alt.flame where they belong.

>Je pense, donc je suis. Et vous?

Likewise, I'm sure. :-)


Stephanie Caruana
Spear Shaker Review - Oxfordian Quarterly On-Line Magazine
http://www.spear-shaker-review.com

--
Symposium1 wrote in message
<19990815165353...@ngol04.aol.com>...


>Oh, Stephanie, I've actually been hoping you would shed some light on this
>argument and show up these people who have been picking on you. But this
is so
>ridiculous an argument, even I had to laugh at it.
>
>The idea that the common use of "I", "you", and "the" in French shows any
kind
>of linkage from Oxford to Shakespeare is pretty ludicrous. You and I both
used
>all three words, and we are decidedly not the same person! They happen to
be
>among the most common words in any language.
>
>Now, if Oxford had used a specific error or idiosyncrasy that Shakespeare
had
>repeated and NO ONE else had repeated the same flaw, that might be
something.
>Even still, the flaw might be more common than we could ascertain, and the
>remaining records simply lost due to time and the impermanence of paper.
And
>it still wouldn't rule out the possibility that Shakespeare and Oxford were
in
>cahoots in some fashion other than Shakespeare being a simple straw man.
>

>In article <7p6sbj$lar$1...@nntp6.atl.mindspring.net>, "Stephanie Caruana"

><spear-...@mindspring.com> writes:
>
>>Even so, a number of words/phrases occur in both selections:
>>
>>j'ai (I have)
>>et (and)
>>votre (your)
>>Dieu (God)
>>temps (time)
>>vous (you)
>>je (I)
>>le (the)
>>vous avez (you have)
>>que (what)
>

>Qu'est-ce que vous avez ici? Mon Dieu!

>Je pense, donc je suis. Et vous?

Likewise, I'm sure. :-)

>Etes-vous?
>
>--Ann
>
Stephanie

David L. Webb

unread,
Aug 16, 1999, 3:00:00 AM8/16/99
to
In article <7p98js$125$1...@nntp8.atl.mindspring.net>, "Stephanie Caruana"
<spear-...@mindspring.com> wrote:

[...]


> Obviously, I wasn't trying to prove that this letter written by a
> 13-year-old boy was an early draft of Hamlet! Or that any of the specific
> words were significant.

Then why did you go to the trouble of enumerating carefully all the
words the two texts had in common?

> [...]Please don't


> get me wrong. I was really trying to see whether there was any level of
> "evidence", no matter how trivial, that would satisfy some of these guys'
> silly demands,

I'm really not following you here. You did *not* regard what you posted
as evidence yourself, yet you thought that your interlocutors (whom you've
painted as hostile) *might* be satisfied with it as evidence -- even though
you weren't?!

> and to ask for a little "evidence" from them in return, which
> is never, ever, forthcoming.

Well, although I never wear brown shirts, perhaps you may mistakenly
number me among the school to which you alluded. Can you point to a
*single* instance on which you've asked for reasonable evidence of an
assertion I've made in this forum when I have not obliged by providing the
evidence sought? On the other hand, I've politely asked you at various
times for evidence of Delia Bacon's Oxfordianism, of Ward's reliability in
reporting the dowager Countess of Oxford's "hasty" marriage, and of the
plausibilty of Oxford's authorship of the lyrics to a Beatles song; so far,
no response to any has been forthcoming.



> We have "evidence, " in his own handwriting, signed, that at age 13, Oxford
> knew French, enough to write the French scenes in Henry V.

So did Elizabeth I -- she was fluent, in fact; so what? As far as I
know, *nobody* has seriously questioned the claim that Oxford knew some
French. However, unless Oxford was the only person in England at the time
who knew French, there's no reason to attach any significance whatever to
his command of the tongue.

[...]


> I wish it were possible for Oxfordians to post on this newsgroup without the
> Brown Shirt School of Literary Criticism feeling it their duty to try to
> insult, hassle, abuse and harass us into non-participation.

You interpret a polite request for evidence or a mild critique of your
last post as "harassment"? Have you ever read a Paul Crowley post? Or one
by Patrick (SUSUPPLY)? Or by Roger Stritmatter?



> It might then be possible for others to participate more freely in open
> discussion as well without fear of being flamed for expressing an opinion of
> their own.

> But since the Brown Shirts are always on duty with their billy clubs,

There is no Billy Club, to my knowledge (and if there were, I wouldn't
be a member); by contrast, Oxfordians can choose between the Shakespeare
Oxford Society and the Blue Boar Society! Jesting aside, if you number
Anne among an imaginary group of "brown shirts", you can scarcely have read
many of her posts. They are invariably inoffensive and eminently sensible,
and they rarely have any connection with authorship issues. If even she
found your argument so amusingly naive as to prompt a response, you might
consider the possibility that the fault lies in your post rather than in
its "brown shirt" readers.

> there
> really isn't much opportunity for open exploration of ideas.

If you have ideas that are supported by *evidence*, I for one would
genuinely love to see them. So far, I've seen mostly wild speculation
(such as your "Golden Slumbers" attribution), mostly without evidence. I
do not join groups, and don't care that much who wrote the Shakespeare
canon. I do care about careful reasoning and rational inference, though,
and I fear that you may be mistaking that concern with an authorship
allegiance I do not profess if you number me (or Anne, for that matter)
among a group of "brown shirts".

Let me conclude by repeating a serious proposal that I made in an
earlier post. I'm still waiting for any hard evidence that Ward had any
idea (other than that prompted by fantasy) when the dowager Countess of


Oxford remarried. I'm also still awaiting a citation proving that Delia

Bacon identified Oxford (rather than Bacon or Raleigh) as the leader of the
Shakespeare authorship cabal. Finally, I'm awaiting evidence that the song
"Golden Slumbers" of the Beatles "could have come from deVere's pen," as
you speculated a few weeks ago. If you provide solid evidence that any


*one* of these claims is correct, I'll subscribe to "Spear Shaker Review"

for a year. If you cannot do so, you would be obliged to publish a brief
Editorial in "Spear Shaker Review", to be run in *every issue* for the
ensuing year, acknowledging that the myth of Oxford's mother's "hasty"
marriage is without foundation, as apparently nobody knows when the
marriage actually took place. This is a handsome offer, since I am the
only party to the bet incurring any financial risk; you're merely risking
eating a little crow.

Note that by supplying solid evidence, I mean that you must read Delia
Bacon's book *in its entirety* and decide for yourself whom she views as
the leader of the group; it will not suffice to extract some brief mention
of Oxford from Bacon's incoherent fantasies. You must also explain how it
happens that *you alone* of all the anti-Stratfordians who have read the
book interpreted Delia Bacon as claiming that Oxford was the group's
leader. Even Ogburn characterizes Delia Bacon as follows:

"Conventionally regarded as the pioneer Baconian, she [D. Bacon] in fact
held that the plays were jointly created by Sir Walter Raleigh as poet and
Francis Bacon [...] as philosopher, with contributions by other poets and
dramatists, especially Edmund Spenser."

Evidence for Ward's claim would be a primary document, or some neutral
indication that the dowager countess was already remarried by April 30,
1563, not just Sobran repeating (without evidence) Ogburn's mistake of
repeating (without evidence) Ward's claim. I'll even be generous and
regard the marriage as "hasty" for the purpose of our bet if you can
demonstrate that the countess was remarried by, say, 18 months after the
death of the Earl in August 1562. I have no idea how you might plausibly
present evidence of Oxford's authorship of "Golden Slumbers", since as far
as I know its authorship has never even been questioned, but I'm open to
reasonable evidence.

What do you say? Do we have a deal? Of course, I rather doubt that
anything will come of this. I made a bet with Art Neuendorffer long ago
that the Shakespeare manuscripts were not buried in Combe's tomb, but so
far he has neither produced the manuscripts nor payed up. But I expect
better things of you than of Art -- you're the editor of a serious
publication, while Art, as I think you've realized, is just trolling.

Finally, you might consider the possibility that your facile dismissal
of those who express reasonable disagreement with you as "brown shirts" may
be far more offensive to many readers than even the most pugnacious of Mr.
Nathan posts, which, even with their needless accusations of lying, are
positively innocuous in comparison. For some readers, Fascism is no joke;
it is deadly serious.

David Webb

Stephanie Caruana

unread,
Aug 16, 1999, 3:00:00 AM8/16/99
to
Greg Reynolds wrote:
>
>Thank you. I will be relying on an expert for a translation.
>

I wrote [Oxford's French letter]

>> August 23, 1563
>>
>> Monsieur tres honorable,
>>
>> Monsieur, j'ai recu vos lettres plaines d'humanite et courtoisie, et fort
>> resemblantes a votre grand amour et singulier affection envers moi, comme
>> vrais enfants devement procrees d'une telle mere pour laquelle je me
trouve

>>(snip)

>> It appears to me that this constitutes evidence for the likelihood that
>> Oxford knew enough French to have written these scenes.
>>

>> Now let me ask you the same thing you have asked of me: What EVIDENCE do
>> you have that William Shaxper of STRATFORD knew even enough French to be
>> able to write those scenes in Henry V?
>

>Oh, he was resourceful, and could get it done within seconds--especially if
the
>players knew French. This is not very hard to imagine. Any french speaker
could
>do it. That the audience understood the French indicates this was no big
deal.

Come. come, where's your EVIDENCE? Are you relying on your Imagination?????
Naughty naughty!


>
>> Try to wrap your brain around the
>> idea that citing the play itself is not proof that Shaxper knew French,
>> since the identity of the author is the question at issue.
>

>I didn't cite the play. I'm only staying on the original question. This
>sideline, that Oxford knew French, is beside the point--but kinda fun.
>

>> If you have a hard time getting "a translation from the era," whatever
>> that means, I will be happy to provide one.
>

>Unless my expert is on holiday, I may have a translation very soon. But,
>please, get yours ready and we'll have a showdown. They may both be perfect
and
>identical--or we may learn a lot. Don't post til we are both prepared,
please.
>I'll give you my progress.

Ready when you are. I don't need an "expert." My two years of high school
French will do nicely. That, and the French medals I won in open
competition in the Southeastern states.


>
>'From the era' means just that--my french expert is a historian as well and
>will provide the proper usage for 1563. Any other approach is pointless.

Says you.


>
>
>Looking forward to the 1563 French showdown. Tell me when you are ready.

Whenever.

Stephanie Caruana

Greg Reynolds

unread,
Aug 16, 1999, 3:00:00 AM8/16/99
to

Stephanie Caruana wrote:

> Greg Reynolds wrote:
> >
> >Thank you. I will be relying on an expert for a translation.
>
>

> >>(snip)


>
> >> Now let me ask you the same thing you have asked of me: What EVIDENCE do
> >> you have that William Shaxper of STRATFORD knew even enough French to be
> >> able to write those scenes in Henry V?
>
> >Oh, he was resourceful, and could get it done within seconds--especially if
> the
> >players knew French. This is not very hard to imagine. Any french speaker
> could
> >do it. That the audience understood the French indicates this was no big
> deal.
>
> Come. come, where's your EVIDENCE? Are you relying on your Imagination?????
> Naughty naughty!

No, I was relying on common sense. I don't see what evidence you need. Its only
language--the Queen spoke French fluently and funded the The Chamberlain's Men
by 1594, so there. If the audience was to hear French dialogue--I believe they
understood French to the degree necessary. Do you agree?

> >> Try to wrap your brain around the
> >> idea that citing the play itself is not proof that Shaxper knew French,
> >> since the identity of the author is the question at issue.
> >
> >I didn't cite the play. I'm only staying on the original question. This
> >sideline, that Oxford knew French, is beside the point--but kinda fun.
> >
> >> If you have a hard time getting "a translation from the era," whatever
> >> that means, I will be happy to provide one.
> >
> >Unless my expert is on holiday, I may have a translation very soon. But,
> >please, get yours ready and we'll have a showdown. They may both be perfect
> and
> >identical--or we may learn a lot. Don't post til we are both prepared,
> please.
> >I'll give you my progress.
>
> Ready when you are.

I am not ready yet--I think I'm waiting out a vacation. It will be worth the
wait, I assure you.

> I don't need an "expert." My two years of high school
> French will do nicely. That, and the French medals I won in open
> competition in the Southeastern states.

Its hilarious that you think your high school French is adequate here, but that
Shakespeare, his players, and all his resources would have difficulty.
This is gonna be good.

> >'From the era' means just that--my french expert is a historian as well and
> >will provide the proper usage for 1563. Any other approach is pointless.
>
> Says you.

Say me. I think you will agree.

> >Looking forward to the 1563 French showdown. Tell me when you are ready.
>
> Whenever.

Thanks for waiting, I won't disappoint.

Greg Reynolds


HARRY HILL

unread,
Aug 16, 1999, 3:00:00 AM8/16/99
to
Listen,boneheads! *Anyone* with the slightest will can learn a
forign language! It takes only a modicum of imtelligence.

In article <37B8B1D6...@megsinet.net>, Greg Reynolds <eve...@megsinet.net> writes...

Stephanie Caruana

unread,
Aug 16, 1999, 3:00:00 AM8/16/99
to

Greg Reynolds wrote in message <37B8B1D6...@megsinet.net>...


>
>
>Stephanie Caruana wrote:
>
>> Greg Reynolds wrote:
>> >
>> >Thank you. I will be relying on an expert for a translation.
>>
>>
>> >>(snip)
>>
>> >> Now let me ask you the same thing you have asked of me: What EVIDENCE
do
>> >> you have that William Shaxper of STRATFORD knew even enough French to
be
>> >> able to write those scenes in Henry V?
>>
>> >Oh, he was resourceful, and could get it done within seconds--especially
if
>> the
>> >players knew French. This is not very hard to imagine. Any french
speaker
>> could
>> >do it. That the audience understood the French indicates this was no big
>> deal.

Understanding French is still not a big deal. At least, it appears to be,
to you.

>>But where's your EVIDENCE? Are you relying on your Imagination?????


>> Naughty naughty!
>
>No, I was relying on common sense. I don't see what evidence you need. Its
only
>language--the Queen spoke French fluently and funded the The Chamberlain's
Men
>by 1594, so there. If the audience was to hear French dialogue--I believe
they
>understood French to the degree necessary. Do you agree?

YOUR evidence, is what I need. Where is it? The French in this play is
very simple. Deliberately so, I would think. Yes, I think most of the
audience would understand the French dialogue well enough to understand what
was going on. Besides, the actions in these scenes would be
self-explanatory to most people. I agree to that.
What I don't understand is why you need an expert to translate this
13-year-old boy's letter. Surely you could get a friend to do it, as you
imagine William Shaxper doing so easily;
as you put it,

>Oh, he was resourceful, and could get it done within seconds--

I mean, it's not such a big deal, is it? I think we have agreed on that.


>
>> >> Try to wrap your brain around the
>> >> idea that citing the play itself is not proof that Shaxper knew
French,
>> >> since the identity of the author is the question at issue.
>> >
>> >I didn't cite the play. I'm only staying on the original question. This
>> >sideline, that Oxford knew French, is beside the point--but kinda fun.
>> >
>> >> If you have a hard time getting "a translation from the era,"
whatever
>> >> that means, I will be happy to provide one.
>> >
>> >Unless my expert is on holiday, I may have a translation very soon. But,
>> >please, get yours ready and we'll have a showdown. They may both be
perfect
>> and
>> >identical--or we may learn a lot. Don't post til we are both prepared,
>> please.
>> >I'll give you my progress.
>>
>> Ready when you are.
>
>I am not ready yet--I think I'm waiting out a vacation. It will be worth
the
>wait, I assure you.

I certainly hope so,. I have finished my trasnlation. It only took about
10 minutes, because it is short and the
French is not especially difficult.


>
>> I don't need an "expert." My two years of high school
>> French will do nicely. That, and the French medals I won in open
>> competition in the Southeastern states.
>
>Its hilarious that you think your high school French is adequate here, but
that
>Shakespeare, his players, and all his resources would have difficulty.
>This is gonna be good.

If you think it was so easy for William Shaxper to write French without any
known education, how come you are so desperately waiting for an "expert"
with an MA or PhD in French to translate this itty bitty letter for you?

We'll see. I'm a bit confused here. You seem to be on both sides of the
fence, and apparently you don't even know where the fence is.
On one side, you say that the French in the play is so simple that anyone
(like the audience at the plays) could write it, and anyone could understand
it.
On the other side, I quote a letter to you written by a 13-year-old boy
(admittedly, a natural genius in the field of languages, which you also
don't appear to understand very well although you dimly perceive that
"Shakespeare" may have a little something going for him that is a little bit
out of the ordinary), and you run for the nearest "expert" without whom you
simply can't function at all. How come you can't understand French youself,
since you are an adult I expect and have had plenty of time to learn at
least a smattering? Perhaps your grasp of language, and communication,
isn't everything you would like to believe it is. I will spare you the
embarrassment of going back to your earlier posts to me and quoting those
insults to my intelligence and knowledge that you were throwing around so
freely. (Let bygones be bygones, I always say. Well, sometimes I say that.
Actually, I never say it.)


>
>> >'From the era' means just that--my french expert is a historian as well
and
>> >will provide the proper usage for 1563. Any other approach is pointless.
>>
>> Says you.
>
>Say me. I think you will agree.
>
>> >Looking forward to the 1563 French showdown. Tell me when you are ready.
>>
>> Whenever.
>
>Thanks for waiting, I won't disappoint.

I hope not. Maybe you could find someone with a couple of years of High
School French to translate it for you in the meantime. Otherwise, I'll wait
for your expert to return.

Greg Reynolds

unread,
Aug 16, 1999, 3:00:00 AM8/16/99
to

Stephanie Caruana wrote:

>
> >
> >No, I was relying on common sense. I don't see what evidence you need. Its
> only
> >language--the Queen spoke French fluently and funded the The Chamberlain's
> Men
> >by 1594, so there. If the audience was to hear French dialogue--I believe
> they
> >understood French to the degree necessary. Do you agree?
>
> YOUR evidence, is what I need. Where is it?

Stephanie, I have no idea what evidence you want. Please restate. What?

> The French in this play is
> very simple. Deliberately so, I would think. Yes, I think most of the
> audience would understand the French dialogue well enough to understand what
> was going on. Besides, the actions in these scenes would be
> self-explanatory to most people. I agree to that.
> What I don't understand is why you need an expert to translate this
> 13-year-old boy's letter.

Are you familiar with the OED? Have you ever watched words evolve over time? If
you were familiar with language or history, you would realize the value of an
expert opinion who knows and teaches French, and studies english and French
history. I already know what the letter says (and it shows EO's character as we
know it) but there are specific nuances that I expect to learn. This same
expert translated Peter Farey's poem from LeDoux to Francis Bacon, and of all
the translations we saw here, this expert revealed far and away a more complete
and concise knowledge of what LeDoux was writing.

Again, you will see.
You are judging something you know nothing about.

> Surely you could get a friend to do it, as you
> imagine William Shaxper doing so easily;
> as you put it,
>
> >Oh, he was resourceful, and could get it done within seconds--
>
> I mean, it's not such a big deal, is it? I think we have agreed on that.

Hope you learn something.

> >> It will be worth the wait, I assure you.
>
> I certainly hope so,. I have finished my trasnlation. It only took about
> 10 minutes, because it is short and the
> French is not especially difficult.

> If you think it was so easy for William Shaxper to write French without any


> known education, how come you are so desperately waiting for an "expert"
> with an MA or PhD in French to translate this itty bitty letter for you?

Asked and answered. Patience is a virtue.

> We'll see. I'm a bit confused here. You seem to be on both sides of the
> fence, and apparently you don't even know where the fence is.
> On one side, you say that the French in the play is so simple that anyone
> (like the audience at the plays) could write it, and anyone could understand
> it.

Not my words or thoughts.

> On the other side, I quote a letter to you written by a 13-year-old boy
> (admittedly, a natural genius in the field of languages, which you also
> don't appear to understand very well although you dimly perceive that
> "Shakespeare" may have a little something going for him that is a little bit
> out of the ordinary), and you run for the nearest "expert" without whom you
> simply can't function at all.

Are you trying out for the role of Margaret? I'm adding value is all.

> How come you can't understand French youself,
> since you are an adult I expect and have had plenty of time to learn at
> least a smattering? Perhaps your grasp of language, and communication,
> isn't everything you would like to believe it is. I will spare you the
> embarrassment of going back to your earlier posts to me and quoting those
> insults to my intelligence and knowledge that you were throwing around so
> freely. (Let bygones be bygones, I always say. Well, sometimes I say that.
> Actually, I never say it.)

Listen, girl, you must have a problem with me. Its probably that I straighten
out your nonsense in public. Post any item of mine you consider out of line.
Then get back on topic.

I for one think it is theft of intellectual property to attribute a man's works
to another. I can not find one fact to associate deVere to the canon, and I
have tried many times. That you charge money for your freak show opens the
possibilities that your product on the open market may be freely judged by
consumers. You have been caught in several lies and mistakes on this forum, so
don't cry to me.

And don't expect nice things from those you offend. You are here to sully
Shakspere's name. Deny it?

> Maybe you could find someone with a couple of years of High
> School French to translate it for you in the meantime. Otherwise, I'll wait
> for your expert to return.

As I said, I already translated it, and frankly am wondering why you would want
it known. It does not speak well of the man.

Stephie's Mood Ring,
Greg Reynolds


John W. Kennedy

unread,
Aug 16, 1999, 3:00:00 AM8/16/99
to
Stephanie Caruana wrote:

> I wish it were possible for Oxfordians to post on this newsgroup without the
> Brown Shirt School of Literary Criticism feeling it their duty to try to
> insult, hassle, abuse and harass us into non-participation.

OK, listen up.

Your brain isn't working right. Have it fixed.

Try, just try, to learn even the most elementary rudiments of literary
criticism, historical research, and logic. Until you have, please just
go away. Your moronic rants, when they fail to be funny (such as your
French word list, which had me laughing so hard I nearly fainted from
hyperventilation), are a considerable annoyance.



> It might then be possible for others to participate more freely in open
> discussion as well without fear of being flamed for expressing an opinion of
> their own.

You haven't demonstrated the right to have an opinion.

> But since the Brown Shirts are always on duty with their billy clubs, there


> really isn't much opportunity for open exploration of ideas.

And this particular rant is an obscenity. Go to Hell, bitch. Or Kansas
-- I'm sure they'll appreciate you there. But stop wasting our time
with your inane grafitti.

John W. Kennedy

unread,
Aug 16, 1999, 3:00:00 AM8/16/99
to
MILKY WAY wrote:
>
> I have a feeling he knew all the Romance languages. Which are French,
> Spanish, and Italian.

and Catalan, Provencal, Portuguese, Sardinian, Rumanian, and a few
others.... But there is no evidence that he knew any languages other
than English, French and Latin, and a goodish argumentum a silentio can
be made that he did _not_ know Spanish or Italian.

The rest of your post, apart from the irrelevant biography of Cervantes,
is nonsense.

Richard Nathan

unread,
Aug 17, 1999, 3:00:00 AM8/17/99
to
Stephanie wants to know what evidence we have that William Shakespeare of
Stratford wrote "HENRY V."

Henry V is published in the first folio.

The first folio says that author is William Shakespeare.

The first folio says that the author was a member of the acting company
that put on the plays, a fellow of Heminge and Condell.

He is listed first among the actors who appeared in the plays.

Jonson calls him "Sweet Swan of Avon."

Digges refers to the author's Stratford monument.

Jonson refers to Shakespeare's buskin - a shoe that actors wore.

Heminge and Condell refer to the author as their fellow.

Augustine Phillips, another actor in the company, left money in his will
to his fellow William Shakespare. This was after Oxford was dead.

In his will, William Shakespeare of Stratford left money to Heminge and
Condell and Burbage money to buy rings. These three were all members of
the acting company.

Also, William Shakespeare's will mentions the Blackfriar's gatehouse.
Heminge was also involved in the purchase of the Gatehouse.

There are numerous legal documents that refer to William Shakespeare as
being a member of the King's Men acting company.

There can be no serious doubt that William Shakespeare of Stratford was a
member of the acting company. Even Stephanie portrays the man from
Stratford in her idiotic play about Oxford, although she has the notion
that Shakespeare could have been an illiterate actor performing in the
plays!

So we know that Shakespare of Stratford was an actor in the company.
Heminge and Condell and Jonson and Digges all clearly state in the first
folio that the actor was the author.

Therefore, unless they were all lying as part of some conspiracy, William
Shakespeare of Stratford wrote HENRY V.

This is far better evidence than Stephanie's evidence that Oxford knew
French, and therefore he could have written HENRY V.


Symposium1

unread,
Aug 17, 1999, 3:00:00 AM8/17/99
to
Moi:

>>Oh, Stephanie, I've actually been hoping you would shed some light on this
>>argument and show up these people who have been picking on you

Stephanie:

>Obviously, I wasn't trying to prove that this letter written by a
>13-year-old boy was an early draft of Hamlet! Or that any of the specific

>words were significant. I had about a 200-word sample, on another subject,

>so of course the most common words would be the only matches. Please don't


>get me wrong. I was really trying to see whether there was any level of
>"evidence", no matter how trivial, that would satisfy some of these guys'

>silly demands, and to ask for a little "evidence" from them in return, which
>is never, ever, forthcoming.

It wasn't at all obvious to me that you were joking. Were you joking before in
your other posts? You seemed even so earnest and annoyed in that one.

So, you were trying to see what evidence would satisfy them, and proceeded to
post the silliest you could find rather than that with the most integrity? I'm
afraid you're falling right into their trap. Or is it your trap? So many
traps, not enough shut.

>We have "evidence, " in his own handwriting, signed, that at age 13, Oxford
>knew French, enough to write the French scenes in Henry V.

That narrows it down considerably. How many other people knew
high-school-level French?

>The Stratfordians have 6 "signatures," all spelled differently, on legal
>documents, and never a "Mais oui" among them. They have no "evidence" that
>Shaxper even knew how to spell his own name, much less write a passage in
>French, no matter how elementary.

Legal documents in England weren't executed in French? You're somehow
unfamiliar with the fact that Marlowe and many other people of the period
spelled their own names inconsistently?

>When pushed, they
>immediately run behind the oozing malt sack barricades and mumble that of
>course, Florio was there to help out, and somebody must have known some
>French around there. [Where?] Then there's the French Wigmaker landlord,
>etc. (Maybe he had some French postcards? Oops, no post office, so that's
>out....) That's their "evidence."

It's no more evidence than your silly list of pronouns. Was that your whole
point? I think we can all agree that none of this particular crap is
"evidence," or, everybody's favorite misused word, "proof," and get some sleep.


By the way, "oozing malt sack barricades" was a good line. Not very civil, mind
you, but pretty funny.

>If you'll note, there are a few replies, with the obligatory abuse, floating
>around to that effect right now, but I won't bother to cite them.

The abuse (from a handful of people) doesn't make them wrong any more than
their abuse against you makes you wrong...or right. It just makes them rude
and sad. They know I would prefer if they behaved. But maybe they enjoy being
a little mean, or maybe some of them need it for their egos. People don't
always behave the way we want them to. Ultimately everyone on this newsgroup
has to ask himself or herself: "is what I get from this group worth what I go
through to get it?" Maybe that answer for you is no. There must be an
Oxfordian-only private list somewhere.

>One of them that I just noticed is even "demanding" that I "prove Oxford
>didn't have someone write the letter for him." Demanding?????
>
>Come on, this is "literary discussion?"

Of course not, dear! What little literary discussion around here is usually
about the plays, not about the authorship issue.

>So let me ask (again) any of these brave Strats to present any real evidence
they
>may have hidden among the vast corridors of Shakesperoid "research" to
>bring forth a few words of real "evidence" that William Shaxper of Stratford
>ever wrote a word of French in his life. Maybe they can find some nice
>hitherto unnoticed Collier forgery that will back them up? Oh, he's dead...
>too bad, he has been so useful!

Lack of evidence does not constitute an impossibility. We aren't left with a
blood sample either, I assume that doesn't by necessity mean if we prick him,
he does not bleed.

>I'd say we're ahead so far, "on the evidence." :-)

Because Oxford wrote a letter in French? I'd like the names of everyone who
wrote a letter in French who lived in England between 1592 and 1616, please.
Mon Dieu, I knew it! The Queen wrote the works! Sorry for the flippancy, but
I think someone ought to be taking up the Queen's case here, and no one has.
What fun!

>I wish it were possible for Oxfordians to post on this newsgroup without the
>Brown Shirt School of Literary Criticism feeling it their duty to try to
>insult, hassle, abuse and harass us into non-participation.

Yeah, and then unfortunately you go and beg for retorts by name-calling. Being
neither a Stratfordian nor an Oxfordian, I'd say it's pretty clear what you
*all* want is an argument anyway. You want to be right for your own sakes.
Otherwise, what are you crusading for? For the honor of some guy or other
who's been dead for four hundred years? Same guy who wrote "After my death,
dear love, forget me quite"?

>It might then be possible for others to participate more freely in open
>discussion as well without fear of being flamed for expressing an opinion of
>their own.

Are you (and they) expressing opinions or professing them as facts? I'm not
very much afraid of expressing my opinion.

>But since the Brown Shirts are always on duty with their billy clubs, there
>really isn't much opportunity for open exploration of ideas.

What about the people who disagree with you, and your own antagonistic tactics,
but who aren't calling you names? What color is my shirt? Your own tone
doesn't seem to connote an "open exploration of ideas." I wish it did.

I wish Oxfordianism were sold softly, like some religious groups who, instead
of preaching, just let me see how much happier they are, how much more
meaningful their lives are, in their new-found beliefs. Nobody seems very
happy. Could that be a poor selling point?

>Maybe if we ignore them, they will eventually get bored and wander off to
>alt.flame where they belong.

Hmm. If everybody ignores everyone who disagrees with him, it would be a
boring newsgroup. I agree that civility is to be desired but it goes both
ways. You don't create a civil playing field by saying: they're Brown Shirts
and I wish they'd go away.

I realize you're taking on one splinter at a time in the hopes of felling a big
tree. But I don't think you've earned this splinter yet, let alone the oak.

Further, the playwright's of French is often quite bungled, both in spelling
and in usage. If your young scholar wrote so pleasingly en francais at 13, why
would his plays have so many errors?

>>Je pense, donc je suis. Et vous?

>Likewise, I'm sure. :-)

I thought I asked <etes-vous?> not <et vous?> Eh bien. I didn't win any
French medals.

--Ann

Nigel Davies

unread,
Aug 17, 1999, 3:00:00 AM8/17/99
to
Stephanie wrote:

>Obviously, I wasn't trying to prove that this letter written by a
>13-year-old boy was an early draft of Hamlet! Or that any of the specific
>words were significant. I had about a 200-word sample, on another subject,
>so of course the most common words would be the only matches. Please don't
>get me wrong. I was really trying to see whether there was any level of
>"evidence", no matter how trivial, that would satisfy some of these guys'
>silly demands, and to ask for a little "evidence" from them in return, which
>is never, ever, forthcoming.

You posited Oxford's letter in French as evidence that he could have
written Henry V. It is no such thing. Knowing the French for "and",
"you", "I", etc. has no relevance whatever to the passages of Henry V.
It is reasonable for anyone to question your claim that this is
"evidence" when it clearly isn't.

>The Stratfordians have 6 "signatures," all spelled differently, on legal
>documents, and never a "Mais oui" among them. They have no "evidence" that
>Shaxper even knew how to spell his own name, much less write a passage in
>French, no matter how elementary.

How do you explain "Oxenford"? And "Oxenforde"? And "Oxinford"? And
"Oxinforde"? How do explain de Vere ALWAYS signing his name as the
3-syllable "Oxenforde" (or variants thereof) but Shakespeare's plays all
exclusively referring to any Earl of Oxenforde as "Oxford"? The title
"Earl of Oxenforde" was granted by royal decree spelled with the
3-syllable "Oxenforde". All descendants of the first Earl to be given
that title always, without exception, signed themselves and referred to
themselves as "Oxenforde". But Shakespeare's plays never do. They always
refer to any Oxford character with the 2-syllable "Oxford".

If you wrote a play including one of your ancestors would you make such
a glaring and unnecessary error as to refer to them as 2-syllable
"Carane" though they were always known and always signed themselves as
"Caruana"?



>One of them that I just noticed is even "demanding" that I "prove Oxford
>didn't have someone write the letter for him." Demanding?????

You need to calm down from your paranoia and read the simple, reasonable
enquiry made of you. The question was:

"What evidence do you have that this letter was actually written by
Oxford? You have the temerity to question the authorship of the works
signed by William Shakespeare yet have Oxford holding the pen of this
letter. Are you absolutely sure that the young Earl didn't have this
written for him?"

The words "demanding?????" and "prove" are nowhere in this enquiry
atall. It merely asks you how sure you can be that the letter was not
written for de Vere. You have invented the words "demand" and "prove"
all by yourself.

>Come on, this is "literary discussion?"

Exactly. But you have a very theatrical response to any reasonable
requests made to you to explain your position or present evidence. And
you commit the ultimate offence in literary discussion of putting words
into your interlocuters' mouths that they never said.
______________________________________________________________________________
nda...@emirates.net.ae

volker multhopp

unread,
Aug 17, 1999, 3:00:00 AM8/17/99
to
Nigel Davies wrote:

> The title
> "Earl of Oxenforde" was granted by royal decree spelled with the
> 3-syllable "Oxenforde". All descendants of the first Earl to be given
> that title always, without exception, signed themselves and referred to
> themselves as "Oxenforde". But Shakespeare's plays never do. They always
> refer to any Oxford character with the 2-syllable "Oxford".

The de Veres were referred to as "Oxford", but only signed themselves
as "Oxenforde". Will a play, regardless by whom, refer to them by the
normal "Oxford", or the signatory "Oxenforde"?



> "What evidence do you have that this letter was actually written by
> Oxford? You have the temerity to question the authorship of the works
> signed by William Shakespeare yet have Oxford holding the pen of this
> letter.

Where in the world do you get this stuff from? We have the 6
"signatures" of Shakspere (see
http://users.erols.com/volker/Shakes/WSSigs.htm)-- none of them were
found on any of the works of Shakespeare.

--Volker

gws

unread,
Aug 17, 1999, 3:00:00 AM8/17/99
to
Ann's piece, appended below, gracefully expresses the agnostic view of the
authorship issue, the importance of good manners in posts, and the contentious
righteousness of many Oxfordians. I could not agree more. Heil Stratford . .
. or something like that!

Grey Satterfield -- who despite their conduct still refuses to call the
Oxfordians crack-brained or dishonest -- but has thought about it.

Symposium1 <sympo...@aol.computer> wrote in message
news:19990817004812...@ngol01.aol.com...

Nigel Davies

unread,
Aug 17, 1999, 3:00:00 AM8/17/99
to
volker multhopp wrote:

> Nigel Davies wrote:
>
> > The title
> > "Earl of Oxenforde" was granted by royal decree spelled with the
> > 3-syllable "Oxenforde". All descendants of the first Earl to be given
> > that title always, without exception, signed themselves and referred to
> > themselves as "Oxenforde". But Shakespeare's plays never do. They always
> > refer to any Oxford character with the 2-syllable "Oxford".
>
> The de Veres were referred to as "Oxford", but only signed themselves
> as "Oxenforde". Will a play, regardless by whom, refer to them by the
> normal "Oxford", or the signatory "Oxenforde"?

Well, Justice Stevens would have a field day with this one. A glorious example
of an Oxfordian "might" with not a single fact to support the invention.

The title "Earl of Oxenforde" is granted by royal decree and is used in all
cases by all of the Oxenfordes in all instances as "Oxenforde" (or spelling
variants thereof). Whether Oxenforde is writing to Elizabeth I, Burghley, his
friend Hicks, the Meta Incognita syndicate, anyone, he is known as "Oxenforde".
But, for the Oxfordian cause, there is now a "normal" way of spelling Oxenforde.
The "normal" way for the Earl of Oxenforde to refer to his title Earl of
Oxenforde is apparently "Oxford" though there is no evidence Volker can present
that shows he or any of his antecedents ever referred to themselves in that way.
The Oxenfordes never ever refer to themselves as "Oxford" but de Vere allegedly
writes plays in which they do. For good measure, Oxenforde puts John de Vere,
13th. Earl of Oxenforde, on the losing side at Tewkesbury in R3 and 6H3 where he
never was. None of this adds up. It makes no sense atall. The author's knowledge
of Oxenforde's ancestry is all wrong and his spelling and pronunciation of the
family title is totally inconsistent with how all the Oxenfordes always
identified themselves.

To someone to whom the Oxenforde title meant nothing and to whom portraying John
de Vere as a loser equally meant nothing it all makes sense. For the
self-aggrandising de Vere to be holding the pen using a version of his own title
that no de Vere ever used and completely messing up his own family's ancestry is
utterly ridiculous.

> > "What evidence do you have that this letter was actually written by
> > Oxford? You have the temerity to question the authorship of the works
> > signed by William Shakespeare yet have Oxford holding the pen of this
> > letter.
>
> Where in the world do you get this stuff from? We have the 6
> "signatures" of Shakspere (see
> http://users.erols.com/volker/Shakes/WSSigs.htm)-- none of them were
> found on any of the works of Shakespeare.

My use of the word "signed" was a slip. I meant she was questioning the
authorship of plays and sonnets credited to William Shakespeare whilst seeing no
reason to question the authorship of French text credited to Oxenforde.
______________________________________________________________________
nda...@emirates.net.ae

KQKnave

unread,
Aug 17, 1999, 3:00:00 AM8/17/99
to
In article <19990817004812...@ngol01.aol.com>,
sympo...@aol.computer (Symposium1) writes:

>Yeah, and then unfortunately you go and beg for retorts by name-calling.
>Being
>neither a Stratfordian nor an Oxfordian, I'd say it's pretty clear what you
>*all* want is an argument anyway. You want to be right for your own sakes.
>Otherwise, what are you crusading for? For the honor of some guy or other
>who's been dead for four hundred years? Same guy who wrote "After my death,
>dear love, forget me quite"?

I'm not crusading. I stick around for the interesting history
and commentary about the plays that I get
from the informed members of the group (even you, Ann!).
The research I do to refute Oxfordian fantasies teaches
me more than anything.
The rest of it is just entertainment. The Oxfordians can be
so obstinately stupid that they are funny, and that's just a fact.
So why are YOU arguing with this knucklehead? She's not
interesting enough for me to bother.

Jim


Nigel Davies

unread,
Aug 18, 1999, 3:00:00 AM8/18/99
to
"John W. Kennedy" wrote:

> MILKY WAY wrote:
> >
> > I have a feeling he knew all the Romance languages. Which are French,
> > Spanish, and Italian.
>
> and Catalan, Provencal, Portuguese, Sardinian, Rumanian, and a few
> others.... But there is no evidence that he knew any languages other
> than English, French and Latin, and a goodish argumentum a silentio can
> be made that he did _not_ know Spanish or Italian.

Yet Oxford's Italian servant, Orazio Cogno, reported that Oxford "speaks Italian
well".

As often is the case, the material ostensibly in favour of Oxford (10-month tour
of Italy, the Bulbeck crest, etc.) only end up shooting his candidature
down. It is indeed funny that the alleged author travelled so extensively in
Italy and spoke Italian so well yet the plays show a conspicuous lack of
knowledge about Italy (with its inland towns on the coast) and no more than the
slightest of acquaintances with the Italian language (despite ample opportunity
to show it off in the many plays set in Italy itself).
______________________________________________________________________
nda...@emirates.net.ae


Stephanie Caruana

unread,
Aug 18, 1999, 3:00:00 AM8/18/99
to
Nigel Davies wrote:
>
>As often is the case, the material ostensibly in favour of Oxford (10-month
tour
>of Italy, the Bulbeck crest, etc.) only end up shooting his candidature
>down. It is indeed funny that the alleged author travelled so extensively
in
>Italy and spoke Italian so well yet the plays show a conspicuous lack of
>knowledge about Italy (with its inland towns on the coast) and no more than
the
>slightest of acquaintances with the Italian language (despite ample
opportunity
>to show it off in the many plays set in Italy itself).

As is so often the case, the material which ostensibly shoots Oxford's
candidature down turns out upon closer examination to have missed the foot
completely. A topological map of the area in question reveals that the
towns involved are located well inland, which has given rise to this
particular misunderstanding. At the time Oxford visited these towns, travel
was mostly done via the rivers (in boats, that is), much as the Thames
served London as a main thoroughfare. In Italy, the travel situation by
road was made even more difficult because (a) the roads were poor or
nonexistent; (b) you ran the risk of being eaten for breakfast by bandits
living in the forests which surrounded the towns.

A VERy good case in point would be "The Two Gentlemen of VERona." As a play
by Shaxper of Stratford, the question of sources, influences, priorities,
translations of plot originals, etc., becomes quite vexed. As a play by
Oxford of Castle Hedingham, Essex, and London, these difficulties become as
smooth as the proverbial baby's bottom (no offense meant.)

For those experts among you who have not reread the play recently, I will
recap the plot briefly, with special attention paid to the geography of the
area, the travels of the main characters and the difficulties they
encountered. The play is set in three towns: Verona, Milan, and Mantua. All
three of these town are located in an area known as the Cisalpine Plains, a
roughly triangular lowlying area between the Alps to the north and the
Apennines to the south. As one might expect, all three of these towns are
situated on or near one of the many rivers that flow through northern Italy.
Verona is set on the Adige River, which flows roughly from west to east.
Milan is on a tributary to the Po River, the great west-to-east flowing
river system which drains the Cisalpine Plains area into the Adriatic Sea.
Mantua is on another tributary to the Po.

The geographical relationship between the three towns is crudely expressed
below:

---------^--^ ^ ^
^ ----^-----ALPS--^------^-----^----^----^----------

/
MILAN
VERONA / ADRIATIC SEA
-----
/
\ ______PO r.
MANTUA /
\
_____________ /
^_
\ /
^_
\_____________
APENNINES
^--------
^
^^^^^---^-^^-

^--- _

_


ACT I:
Scene 1. Verona: Valentine and Proteus, the "two gentlemen of Verona":
.Valentine expresses his intent to leave the one-horse town of Verona to
see the world, which in this case means Milan, the big city to the west, via
the Po River
......
VALENTINE: Once more adieu! My father at the road*
Expects my coming, there to see me shipp'd. (1)
......[he leaves]
Enter SPEED, Valentine's servant
SPEED....Saw you my master?
PROTEUS: But now he parted hence, to embark for Milan. (2).

*I take this to mean river-way.
(1) Text and all references are drawn from The Complete Works of
Shakespeare, Hardin Craig, ed., 1961 edition. This is the first reference
in the play to traveling by boat, one of those old Stratfordian chestnuts
still trotted out by some Stratfordians, as cited above. Oddly enough, even
this 1961 edition was aware that the text cited was not inherently ludicrous
or necessarily indicative of ignorance upon the part of the author. NOTE to
above lines: "To Milan: Today one does not travel from Verona to Milan by
ship. In the sixteenth century this may have been the easier route." Craig,
p.133.

(2) See Note 1.


^-^-^-^-^^---------^-----------------^------^^--^^-----^^^^-------------^^^^
^-

Well, obviously this post can't continue much longer, but I do believe I
will continue this and use it as an article in the current issue of Spear
Shaker Review as a special addenda. It will be up there within one week.
Just using the Craig edition as a source for the conventional analysis of
this play, its sources, etc., I don't see how there would be anyone left
who would attempt to maintain that Shaxper was "obviously" the author of
this play, and Oxford was "obviously" not the author.

Just as a sporting proposition, I make this offer to participants of HLAS:

FREE ONE-MONTH TRIAL SUBSCRIPTION TO SPEAR SHAKER REVIEW

If, within the next seven days, you send me an E-mail requesting a free
trial subscription, I will send you by return E-mail an ID and Password that
will be valid for one month. At the end of the month-long trial period,
this ID and Password will turn into a pumpkin and you will no longer have
access to the Spear Shaker Review site. You can then, of course, subscribe
to Spear Shaker Review at the regular subscription rate of $20 per year.

EXCEPTION: Anyone who sends me a useable article or letter in response to
this or any other article, or a new article on a different topic accepted
for publication in Spear Shaker Review, will receive a free one-year
subscription. This applies to participants of every stripe: Stratfordians,
Oxfordians, Baconians, Marlovians, Rutlandians, Unicornians, etc. Humor
will not be considered a drawback. Articles which essentially say,
"Shakespeare wrote the plays because it says so in the Folio" will not be
considered.

This offer of a free one-month trial subscription will extend from 12:00
noon on August 17 to 12:00 midnight on August 24, 1999.

The trial subscriiption period will run from today, August 17th, through
August 31, 1999.

Thank you for your interest and participation.


Stephanie Caruana
Spear Shaker Review - On-line Quarterly Oxfordian Magazine
http://www.spear-shaker-review.com

http://dmoz.org/arts/humanities/english_literature/shakespeare/authorship/

Nigel Davies wrote in message <37BA7252...@emirates.net.ae>...

I find it indeed funny that you think the author would feel a need to show
off his knowledge of Italian by using Italian in the plays.
>______________________________________________________________________
>nda...@emirates.net.ae
>

gws

unread,
Aug 18, 1999, 3:00:00 AM8/18/99
to
John W. Kennedy <rri...@ibm.net> wrote in message
news:37B89ECC...@ibm.net...

| Stephanie Caruana wrote:
|
| > I wish it were possible for Oxfordians to post on this newsgroup without
the
| > Brown Shirt School of Literary Criticism feeling it their duty to try to
| > insult, hassle, abuse and harass us into non-participation.
|
| OK, listen up.
|
| Your brain isn't working right. Have it fixed.
|
| Try, just try, to learn even the most elementary rudiments of literary
| criticism, historical research, and logic. Until you have, please just
| go away. Your moronic rants, when they fail to be funny (such as your
| French word list, which had me laughing so hard I nearly fainted from
| hyperventilation), are a considerable annoyance.
|
| > It might then be possible for others to participate more freely in open
| > discussion as well without fear of being flamed for expressing an opinion
of
| > their own.
|
| You haven't demonstrated the right to have an opinion.

|
| > But since the Brown Shirts are always on duty with their billy clubs,
there
| > really isn't much opportunity for open exploration of ideas.
|
| And this particular rant is an obscenity. Go to Hell, bitch. Or Kansas
| -- I'm sure they'll appreciate you there. But stop wasting our time
| with your inane grafitti.

Oh, spit it out John. Call a spade a G__ D___ spade, and not a spadey wadey!
I might have stated John's sentiments with a little more restraint than he
showed. Nevertheless, I admit that Stephanie's posts do seem to have the
ability to bring out the worst in others.

Grey Satterfield

Stephanie Caruana

unread,
Aug 18, 1999, 3:00:00 AM8/18/99
to

A couple of mistakes were made in the dates listed in my previous post.
The dates for the beginning and ending of the 1-week period for obtaining a
free one month subscription to Spear Shaker Review should have read:

This offer of a free one-month trial subscription will extend from 12:00

>noon on August 18 to 12:00 midnight on August 25, 1999.

instead of:

This offer of a free one-month trial subscription will extend from 12:00
>noon on August 17 to 12:00 midnight on August 24, 1999.

The dates for the free tiral subscription period should have read:
>The free trial subscription period will run from today, August 18th,
through
>September 30, 1999.

instead of:

>The trial subscriiption period will run from today, August 17th, through
>August 31, 1999.

Also, unfortunately the map I drew got disarranged so it doesn't make much
sense in its Net configuration, but a more clearpicture will be available on
SSR.

> ^^------^^^-^-- ^ ----^-----ALPS--^------^-----^----^----^----------
>
/
>
/
> Milan
Verona / Adriatic
> |
| /
\ Mantua
/
> \_______Po river_________|___________/
> .
/

> ^---^
^-^^^^---
> ^--^-- APENNINES ^^---
> ^--------

>noon on August 18 to 12:00 midnight on August 25, 1999.
>
>The trial subscriiption period will run from today, August 18th, through
>September 30, 1999.


>
>Thank you for your interest and participation.
>
>

volker multhopp

unread,
Aug 18, 1999, 3:00:00 AM8/18/99
to
Stephanie Caruana wrote:

> Just as a sporting proposition, I make this offer to participants of HLAS:

> FREE ONE-MONTH TRIAL SUBSCRIPTION TO SPEAR SHAKER REVIEW

> If, within the next seven days, you send me an E-mail requesting a free
> trial subscription, I will send you by return E-mail an ID and Password that
> will be valid for one month. At the end of the month-long trial period,
> this ID and Password will turn into a pumpkin and you will no longer have
> access to the Spear Shaker Review site. You can then, of course, subscribe
> to Spear Shaker Review at the regular subscription rate of $20 per year.

I'll gladly give it a trial run.

--Volker

Symposium1

unread,
Aug 19, 1999, 3:00:00 AM8/19/99
to
In article <19990817191432...@ngol02.aol.com>,
kqk...@aol.comspamslam (KQKnave) writes:

>>Yeah, and then unfortunately you go and beg for retorts by name-calling.
>>Being neither a Stratfordian nor an Oxfordian, I'd say it's pretty clear what
you
>>*all* want is an argument anyway. You want to be right for your own sakes.
>>Otherwise, what are you crusading for? For the honor of some guy or other
>>who's been dead for four hundred years? Same guy who wrote "After my death,
>>dear love, forget me quite"?

>I'm not crusading. I stick around for the interesting history
>and commentary about the plays that I get
>from the informed members of the group (even you, Ann!).
>The research I do to refute Oxfordian fantasies teaches
>me more than anything.

Clearly, I over-wrote my point and enlarged my comment to wide to embrace
everyone. I was mainly addressing Stephanie and her "Brown-Shirts".

>The rest of it is just entertainment. The Oxfordians can be
>so obstinately stupid that they are funny, and that's just a fact.

Oh, I think we can all be a little 'funny' from time to time. Especially me.

>So why are YOU arguing with this knucklehead? She's not
>interesting enough for me to bother.

I just thought the list of French words was funny, and was totally surprised to
see her comeback was "gee, you didn't think I was serious...did you?" After
that I felt on the receiving end of a whole bunch of venting. So I figured,
without calling her an idiot, I would point out to her that her arguments did
indeed ring as inconclusive -- even to one not intently bent on destroying the
Oxfordian argument.

I like to think that I'm somewhat open to any serious claim of alternative
authorship, and I'm certainly not well educated. So presumably it wouldn't be
too hard to convince me, and I'm not convinced.

And I just wanted to express my opinion, Jim, that brown shirts or no brown
shirts, expressing a feeling of being persecuted doesn't lend one's argument
any more credibility. Not even from me, the nancy politically correct pacifist.
Which, I think, runs contrary to the scheme employed in Ogburn's gigantic
book. I felt he was trying very hard to make me feel sorry for him, about how
badly the literary establishments had treated him. Apparently he didn't think
this was distracting from his other point, but I found it to be so.

--Ann

Nigel Davies

unread,
Aug 19, 1999, 3:00:00 AM8/19/99
to
Stephanie Caruana wrote:

>I find it indeed funny that you think the author would feel a need to show
>off his knowledge of Italian by using Italian in the plays.

Of course. That never occurred to me before. Why would anyone who spoke Italian
so well, travel extensively in Italy, write plays set in Italy, and actually use
the Italian language in them? He uses rudimentary French in a play set in
France, Latin in various places, but why should he use Italian in plays set in
Italy? How ridiculous of me to expect that.

Perhaps this is all part of the Oxinforde conspiracy. If he had used Italian to
any appreciable degree in the Italian plays then everyone would have suspected
the Earl as their author so he deliberately avoided doing so? Yeah, sure.
______________________________________________________________________
nda...@emirates.net.ae

Nigel Davies

unread,
Aug 19, 1999, 3:00:00 AM8/19/99
to
Stephanie Caruana wrote:

> > As is so often the case, the material which ostensibly shoots Oxford's
> >candidature down turns out upon closer examination to have missed the foot
> >completely. A topological map of the area in question reveals that the
> >towns involved are located well inland, which has given rise to this
> >particular misunderstanding. At the time Oxford visited these towns,
> travel
> >was mostly done via the rivers (in boats, that is), much as the Thames
> >served London as a main thoroughfare.

Errr, sorry, but you need to present some evidence to support this invention. It
is a well documented fact that the Grand Tour of Italy conducted by the English
aristocracy and alike was principally by road, not rivers. It was the Grand
Tour, not the Grand Cruise. The tortuous logistics of travelling between these
cities by boat or ship is a main reason why they went by road. Giovanni
Battista's itinerary of the Italian Grand Tour in 1563, for example,
demonstrates this amply. Can you provide the itinerary of anyone who visited
Italian cities other than Venice who did so "mostly via the rivers in boats" as
you claim? I'm not aware of even one.

Your Thames analogy is also ridiculous. We're talking about the common means of
travel between major dispersed cities in Northern Italy, not one major waterway
serving one major English city.

It also begs the question why de Vere would have been motivated to write any
plays about Italy on his return as he was evidently not too impressed with it.
In his own words: "for my lekinge of Italy, my lord I am glad I haue sene it,
and I care not euer to [se] see it any more vnles it be to serue my prince or
contrie". He literally wouldn't care if he never saw Italy again.

> In Italy, the travel situation by
> >road was made even more difficult because (a) the roads were poor or
> >nonexistent;

Really? Where on earth did you get this nonsense from? The great road
engineering innovators of Europe, the Romans, never existed did they? Milan and
Verona were major Italian cities at the time, as they are now. The trade between
the two was extensive. They have been excellently serviced by ancient and
well-travelled roads for millennia. It's a mere 90 miles between these 2 cities.
Verona is strategically located on the main route from Milan to Venice and from
Italy to central Europe via the Brenner Pass which has had excellent roads in
use since Roman times.

It's amazing that you have people travelling from Verona to Milan, via a complex
of contra-flowing rivers, on one of the most tortuous voyages of all time, that
must well exceed 1,000 miles, and taken weeks to complete, when Milan is
literally just 90 miles down the main road from Verona: a 2 or 3 day journey.

> >(b) you ran the risk of being eaten for breakfast by bandits
> >living in the forests which surrounded the towns.

The evidence of those who conducted the Grand Tour by road is tangible evidence
that this is a fable even Aesop would have rejected at first draft.

> >A VERy good case in point would be "The Two Gentlemen of VERona." As a
> play
> >by Shaxper of Stratford, the question of sources, influences, priorities,
> >translations of plot originals, etc., becomes quite vexed.

Huh? Care to be specific. There's nothing atall vexed about it atall.

> As a play by
> >Oxford of Castle Hedingham, Essex, and London, these difficulties become as
> >smooth as the proverbial baby's bottom (no offense meant.)

Au contraire. It's the most hilarious, unbelievable, tortuous, road-vanishing,
current-reversing, epic voyage inventing, debilitating invention ever.

> >For those experts among you who have not reread the play recently,

What about the other plays as well as 2GoV? Like The Tempest that depicts Milan
as a direct sea-port? TotS that has a sailmaking business in the town of Bergamo
that's in the foothills of the Italian Alps; Padua depicted as a sea-port; Padua
in the region of Lombardy which it has never been a part of?

> I will
> >recap the plot briefly, with special attention paid to the geography of the
> >area, the travels of the main characters and the difficulties they
> >encountered. The play is set in three towns: Verona, Milan, and Mantua. All
> >three of these town are located in an area known as the Cisalpine Plains, a
> >roughly triangular lowlying area between the Alps to the north and the
> >Apennines to the south. As one might expect, all three of these towns are
> >situated on or near one of the many rivers that flow through northern
> Italy.
> >Verona is set on the Adige River, which flows roughly from west to east.

in the completely opposite direction to Milan which lies west of Verona.

De Vere visited Italy for 10 months, and clearly did not travel between these
cities in the tortuous fashion you suggest and clearly would not have put such a
tortuous voyage in a play when it is a matter of recorded fact that he visited
at least Milan, presumably with his eyes open, to see that Verona was just 90
miles away on good and well-travelled commercial roads.

> >Milan is on a tributary to the Po River, the great west-to-east flowing
> >river system

again, in the opposite direction ideally required for a sailing vessel to travel
from east to west from Verona to Milan.

Neither did they then. They'd been travelling between these towns since at least
Roman times via perfectly good, commercial roads.

> In the sixteenth century this may have been the easier route."

Well, why present evidence of anyone ever actually travelling by ship between
these cities in the 16th. Century when (like Justice Stevens points out)
Oxfordians can just lean back on "might" or "may" to prop up their case?
______________________________________________________________________
nda...@emirates.net.ae

Stephanie Caruana

unread,
Aug 19, 1999, 3:00:00 AM8/19/99
to


Nigel Davies wrote in message <37BBCE6A...@emirates.net.ae>...


>Stephanie Caruana wrote:
>
>>I find it indeed funny that you think the author would feel a need to show
>>off his knowledge of Italian by using Italian in the plays.
>

>Of course. That never occurred to me before. Why would anyone who spoke
Italian
>so well, travel extensively in Italy, write plays set in Italy, and
actually use
>the Italian language in them? He uses rudimentary French in a play set in
>France, Latin in various places, but why should he use Italian in plays set
in
>Italy? How ridiculous of me to expect that.

Maybe you would have liked to show off that way, but apparently
"Shakespeare", whoever he was, did not.

It was your idea that a man so brilliant and accomplished, who was writing
plays presented at Court for the Queen, etc., and participating fully in
Court life, would feel a need to "show off" his knowledge of the Italian
language [for the sake of showing off] that struck me as "funny.". He
certainly revealed his knowledge of Italian, as well as of Italy and Italian
and French literature, in these plays set in Italy, but he was writing for
an English-speaking audience, to entertain them. When Elizabeth wanted to
hear a play or speech in Latin, she got one. But it probably wasn't a load
of laughs, in the same sense as "Taming of the Shrew" must have been.
"Shakespeare" used French in Henry V as it was appropriate. The "English
lesson" scene is cute, and shows the French princess trying to get ready to
deal with Henry V, by learning a few words of English, in case he wins the
victory over the French which he did win. In the second scene in which
French is used, the scene on the battlefield where Pistol is trying to
extort a ransom from a French prisoner with the aid of a Boy who is
interpreting, Shakespeare introduces the bloody battle to come. It's also
perhaps worth noting that the French prisoner's name is "Fer," (and that
someone named "Fer" is named, but never appears, in another of the plays).
This scene has another hidden dimension of irony, in that Pistol has made a
deal with the Frenchman Fer to spare his life in the return for the promised
ransom, which the Boy has helped to arrange. Pistol and "Master Fer" exit,
leaving the Boy alone. The Boy talks about Bardolph and Nym, Pistol's old
cronies, mentions that they are hanged, speculates on what will become of
Pistol, and then runs off to help guard the baggage train. By the end of the
play, both the Boy and "Master Fer" are dead; the Boy has been killed when
the French massacre the boys who are guarding the baggage train, and "Master
Fer" has been slaughtered by Pistol by ordered of Henry V, in retaliation
for the massacre of all the boys who were guarding the baggage train. It's
a beautiful bit of playwriter's craft, to show in this "comic" scene
representatives of 2 groups of people who are soon to die, and in that way
humanize them; otherwise, the news of the deaths of two groups of people
would not be as meaningful. This is the way French is used here; certainly
not to "show off."


>Perhaps this is all part of the Oxinforde conspiracy. If he had used
Italian to
>any appreciable degree in the Italian plays then everyone would have
suspected
>the Earl as their author so he deliberately avoided doing so? Yeah, sure.

This statement is too silly for comment.
>______________________________________________________________________
>nda...@emirates.net.ae
>
>

KQKnave

unread,
Aug 19, 1999, 3:00:00 AM8/19/99
to
In article <19990819025138...@ngol08.aol.com>,
sympo...@aol.computer (Symposium1) writes:

> Which, I think, runs contrary to the scheme employed in Ogburn's gigantic
>book. I felt he was trying very hard to make me feel sorry for him, about
>how
>badly the literary establishments had treated him. Apparently he didn't
>think
>this was distracting from his other point, but I found it to be so.
>

I didn't notice that but I didn't read every turgid word. To me
it seemed like he was very cynically misrepresenting the facts
in order to make a sensational claim that would earn him some
money, like the people who write about the so-called Bermuda
Triangle and UFO's etc.


Jim


John W. Kennedy

unread,
Aug 19, 1999, 3:00:00 AM8/19/99
to
Nigel Davies wrote:
>
> Errr, sorry, but you need to present some evidence to support this invention. It
> is a well documented fact that the Grand Tour of Italy conducted by the English
> aristocracy and alike was principally by road, not rivers. It was the Grand
> Tour, not the Grand Cruise. The tortuous logistics of travelling between these
> cities by boat or ship is a main reason why they went by road. Giovanni
> Battista's itinerary of the Italian Grand Tour in 1563, for example,
> demonstrates this amply. Can you provide the itinerary of anyone who visited
> Italian cities other than Venice who did so "mostly via the rivers in boats" as
> you claim? I'm not aware of even one.

I don't recall Cellini mentioning any river travel, either.

Nigel Davies

unread,
Aug 20, 1999, 3:00:00 AM8/20/99
to
Stephanie Caruana wrote:

> Nigel Davies wrote in message <37BBCE6A...@emirates.net.ae>...
> >Stephanie Caruana wrote:
> >
> >>I find it indeed funny that you think the author would feel a need to show
> >>off his knowledge of Italian by using Italian in the plays.
> >
> >Of course. That never occurred to me before. Why would anyone who spoke
> Italian
> >so well, travel extensively in Italy, write plays set in Italy, and
> actually use
> >the Italian language in them? He uses rudimentary French in a play set in
> >France, Latin in various places, but why should he use Italian in plays set
> in
> >Italy? How ridiculous of me to expect that.
>
> Maybe you would have liked to show off that way, but apparently
> "Shakespeare", whoever he was, did not.

I think it's more a case of "could not" than "did not". Shakespeare was a magpie
of a writer including so many facets of life from law to music to language and
beyond. French narrative in a play set in France is entirely to be expected.
Latin narrative in other plays (including Roman ones) from a man with a Latin
Grammar education is entirely to be expected. Italian narrative in the Italian
plays, I contend, is also entirely to be expected. But it's not there. It's not
that the records have been lost, or that we're not sure whether de Vere could
actually speak Italian: the canon is remarkably bereft of any demonstration of
competence in Italian despite your candidate speaking the language so well;
having spent 10 months touring the country; having taken home an Italian
national as his "house boy"; the Italian plays providing the perfect setting for
"choice Italian" to be included; and the author of the plays having a
demonstrable habit of including national languages in the plays that he could.

>From an objective, dispassionate, rational, non-jaundiced,
let's-not-colour-our-judgement-with-prejudices point of view, this is indicative
of a lack of linguistic competence in Italian that jars with de Vere's
house-boy's testimony of him speaking the language well.

And this carries through into the Sonnets: sonnets with the subject-matter of
Italy, the house he built in Italy, his travels around Italy, etc.? Zippo.

Then again, perhaps Shakespeare had de Vere in mind when he has Portia
ridiculing English, Scottish and German nobility abroad, with the English
aristocrat Falconbridge having "neither Latin, French, nor Italian", being "a
poor pennyworth in the English", and being "a dumb show".

> It was your idea that a man so brilliant and accomplished, who was writing
> plays presented at Court for the Queen, etc., and participating fully in
> Court life, would feel a need to "show off" his knowledge of the Italian
> language [for the sake of showing off] that struck me as "funny.".

Stephanie, de Vere was one of the most self-aggrandising human beings that DNA
has ever produced. Edward de Vere's number one priority in life was the
indulgence and promotion of Edward de Vere. From hedonistic fetishes with an
Italian house-boy to annual lobbying for the Order of the Garter nothing drove
him more than self-aggrandisement and self-indulgence. Had such a personality
the opportunity to flaunt his knowledge of Italian, let alone the basic
opportunity that the Italian plays presented and the expectation for them to
include Italian, it's inconceivable that such a man would not have used it.

> He certainly revealed his knowledge of Italian,

In what way? The canon shows the remotest of acquaintances with the Italian
language.

> as well as of Italy

Like putting Milan on the coast and putting Padua in the region of Lombardy to
complement giving landlocked Bohemia a coast?

> and Italian and French literature,

So did hundreds of other literary figures in the Rennaisance.

> in these plays set in Italy, but he was writing for
> an English-speaking audience, to entertain them. When Elizabeth wanted to
> hear a play or speech in Latin, she got one. But it probably wasn't a load
> of laughs, in the same sense as "Taming of the Shrew" must have been.
> "Shakespeare" used French in Henry V as it was appropriate. The "English
> lesson" scene is cute, and shows the French princess trying to get ready to
> deal with Henry V, by learning a few words of English, in case he wins the
> victory over the French which he did win. In the second scene in which
> French is used, the scene on the battlefield where Pistol is trying to
> extort a ransom from a French prisoner with the aid of a Boy who is
> interpreting, Shakespeare introduces the bloody battle to come. It's also
> perhaps worth noting that the French prisoner's name is "Fer," (and that
> someone named "Fer" is named, but never appears, in another of the plays).

I'm not aware of any character called "Fer" appearing in any other play than H5.

> This scene has another hidden dimension of irony, in that Pistol has made a
> deal with the Frenchman Fer to spare his life in the return for the promised
> ransom, which the Boy has helped to arrange. Pistol and "Master Fer" exit,
> leaving the Boy alone. The Boy talks about Bardolph and Nym, Pistol's old
> cronies, mentions that they are hanged, speculates on what will become of
> Pistol, and then runs off to help guard the baggage train. By the end of the
> play, both the Boy and "Master Fer" are dead; the Boy has been killed when
> the French massacre the boys who are guarding the baggage train, and "Master
> Fer" has been slaughtered by Pistol by ordered of Henry V, in retaliation
> for the massacre of all the boys who were guarding the baggage train. It's
> a beautiful bit of playwriter's craft, to show in this "comic" scene
> representatives of 2 groups of people who are soon to die, and in that way
> humanize them; otherwise, the news of the deaths of two groups of people
> would not be as meaningful. This is the way French is used here; certainly
> not to "show off."
>
> >Perhaps this is all part of the Oxinforde conspiracy. If he had used
> Italian to
> >any appreciable degree in the Italian plays then everyone would have
> suspected
> >the Earl as their author so he deliberately avoided doing so? Yeah, sure.
>
> This statement is too silly for comment.

Actually, it's the level of Oxinfordian scholarship demonstrated on this list.
One of your peers has claimed that Oxford put Padua on the coast in his plays
because "he obviously had it in for someone from Padua". I'm sure that Paduan he
had it in for must have been mortally wounded by such a move. Another has
insisted that de Vere was an actor who used to limp across the stage in heavy
make-up to disguise his identity from the other actors and miraculously none of
his co-actors nor the audience ever suspected anything funny about this "painted
monster". The same one devised an elaborate scenario of de Vere being entranced
in his nursery by the Bulbeck crest and going on to use it as his inspiration
for the name "Shake-speare" in blissful ignorance that the Bulbeck crest never
had anything to do with de Vere nor that branch of his family. One even
said 'Golden Slumbers' "struck me as a particularly beautiful lyric, so
I remembered it. When I heard Paul McCartney sing it, I realized that he must
have liked the lyric too, and wrote a little melody for it. Therefore as far as
I am concerned, 'Golden Slumbers' could have come from DeVere's pen."
______________________________________________________________________
nda...@emirates.net.ae


gws

unread,
Aug 20, 1999, 3:00:00 AM8/20/99
to

KQKnave <kqk...@aol.comspamslam> wrote in message
news:19990819133334...@ngol01.aol.com...

I am inclined to subscribe to both analyses. The sine qua non of Ogburn, it
seems to me, is its essential goofiness. I didn't read every turgid word,
either, I can't imagine how anyone could -- with the possible exception of
Ogburn himself, his close relatives, and , possibly, his editor.

Grey Satterfield

Paul Crowley

unread,
Aug 20, 1999, 3:00:00 AM8/20/99
to
On Thu, 19 Aug 1999 09:08:37 -0400, "Stephanie Caruana"
<spear-...@mindspring.com> wrote:

>Nigel Davies wrote in message <37BBCE6A...@emirates.net.ae>...


>>Of course. That never occurred to me before. Why would anyone who spoke
>>Italian so well, travel extensively in Italy, write plays set in Italy, and
>>actually use the Italian language in them?

> When Elizabeth wanted to


>hear a play or speech in Latin, she got one. But it probably wasn't a load
>of laughs, in the same sense as "Taming of the Shrew" must have been.
>"Shakespeare" used French in Henry V as it was appropriate.

The answer must be that Elizabeth's knowledge of Italian was
weak. (While she had received instruction from Castiglione in
her youth, it was never a language that she mastered.) She would
probably just have been irritated by passages in Italian, or by
the use of Italian words. Whereas she was fluent in French and
Latin. And the bulk of the court would have followed suit, more
or less. While many would know some Italian, it was not a
'compulsory subject' as were French and Latin.

Paul.

Nigel Davies

unread,
Aug 22, 1999, 3:00:00 AM8/22/99
to
Paul Crowley wrote:

> >Nigel Davies wrote in message <37BBCE6A...@emirates.net.ae>...
> >>Of course. That never occurred to me before. Why would anyone who spoke
> >>Italian so well, travel extensively in Italy, write plays set in Italy, and
> >>actually use the Italian language in them?
>
> > When Elizabeth wanted to
> >hear a play or speech in Latin, she got one. But it probably wasn't a load
> >of laughs, in the same sense as "Taming of the Shrew" must have been.
> >"Shakespeare" used French in Henry V as it was appropriate.
>
> The answer must be that Elizabeth's knowledge of Italian was
> weak. (While she had received instruction from Castiglione in
> her youth, it was never a language that she mastered.) She would
> probably just have been irritated by passages in Italian, or by
> the use of Italian words.

Nice invention, Paul. But contradicted by the facts, as usual. Catherine
Parr ensured Elizabeth received a
rigorous education in languages by way of the formidable tutor, Roger
Ascham, who reported that she was fluent in Greek, Latin, French, and
Italian.
______________________________________________________________________________
nda...@emirates.net.ae

Greg Reynolds

unread,
Aug 24, 1999, 3:00:00 AM8/24/99
to
Stephanie Caruana wrote:

>A couple of mistakes were made in the dates listed in my previous
>post. The dates for the beginning and ending of the 1-week period
>for obtaining a free one month subscription to Spear Shaker Review
>should have read:

>This offer of a free one-month trial subscription will extend from
>12:00 noon on August 18 to 12:00 midnight on August 25, 1999.

Hi Stephanie. I'm in.
Greg Reynolds


robert...@my-deja.com

unread,
Aug 25, 1999, 3:00:00 AM8/25/99
to
In article <37C384CA...@megsinet.net>,
Greg Reynolds <eve...@megsinet.net> wrote:

> Stephanie Caruana wrote:
>
> >A couple of mistakes were made in the dates listed in my previous
> >post. The dates for the beginning and ending of the 1-week period
> >for obtaining a free one month subscription to Spear Shaker Review
> >should have read:
>
> >This offer of a free one-month trial subscription will extend from
> >12:00 noon on August 18 to 12:00 midnight on August 25, 1999.
>
> Hi Stephanie. I'm in.
> Greg Reynolds
>

You can put me down for the free one-month subscription,
too, Stephanie--but only if you agree not to be upset
when I don't extend it--which I will have to do regardless
of whether I like it or not because I am incredibly broke,
and will be for some time.

--Bob G.
>


Sent via Deja.com http://www.deja.com/
Share what you know. Learn what you don't.

Stephanie Caruana

unread,
Aug 25, 1999, 3:00:00 AM8/25/99
to
No problem, Bob. Thanks for taking up my offer.

Stephanie

--


Stephanie Caruana
Spear Shaker Review - On-line Quarterly Oxfordian Magazine
http://www.spear-shaker-review.com

http://dmoz.org/arts/humanities/english literature/shakespeare/authorship

robert...@my-deja.com wrote in message <7q1c4c$6hi$1...@nnrp1.deja.com>...


>In article <37C384CA...@megsinet.net>,
> Greg Reynolds <eve...@megsinet.net> wrote:
>> Stephanie Caruana wrote:
>>

>> >A couple of mistakes were made in the dates listed in my previous
>> >post. The dates for the beginning and ending of the 1-week period
>> >for obtaining a free one month subscription to Spear Shaker Review
>> >should have read:
>>
>> >This offer of a free one-month trial subscription will extend from
>> >12:00 noon on August 18 to 12:00 midnight on August 25, 1999.
>>

Marlovian

unread,
Aug 26, 1999, 3:00:00 AM8/26/99
to
Guess I oughta take up the offer before the deadline. Thanks.
David More

john_baker

unread,
Oct 3, 1999, 3:00:00 AM10/3/99
to
On Mon, 09 Aug 1999 22:52:22 -0600, Dave Kathman <dj...@ix.netcom.com>
wrote:


Boy its really a scream to see David Kathman claiming that
Shakespeare may have known some French or that he might have picked it
up from staying with the Mountjoys. Dave must know he roomed with
this family AFTER he wrote the Henriad. Even SS figured this out.

Scholars know that Sk, whoever he was, read Latin, Greek, French,
Italian and Spanish. He very likely read Hebrew, Arabic, Dutch and
German as well. Or as they said of HS "known in all lanauges..."

There's lots of info on SK's reading habits if you aren't stuck on the
crazy notion that he was that fool the actor, you know the guy you
claimed in his will that after his "deseas" his illiterate daughters
would be the "extetoes" of his estate "for the betr enablin of her to
pfrom" his wishes.

Sure David, thanks for the help.

In H4 he has Falstaff reading Galen's Progonis...and later translates
from it himself....not what you'd call casual materials. You need to
check out the topic on the board about Sk's Italian, I'll include it
in a post on this topic below.

baker
>M.R.Kohanna wrote:
>>
>> What foreign languages did Shakespeare know, if any?
>>
>>David Kathman wrote:

>> I'm sure he knew some Latin, but was he fluent in French or any other language?
>> Thanks for any help.
>
>I don't know how fluent he was, but he certainly knew some French, as
>evidenced by Henry V and a few other plays. Shakespeare lived with a
>French family for a while, and a significant source for his
>early-to-middle
>plays and poems (written 1593-8) is John Eliot's French phrase book
>*Orthoepia Gallica*, published by Shakespeare's fellow Stratfordian
>Richard Field in 1593. Eliot was also from Warwickshire, so the three
>men could have even known each other before 1593.
>
>Dave Kathman
>dj...@ix.netcom.com


John W. Kennedy

unread,
Oct 4, 1999, 3:00:00 AM10/4/99
to
john, baker wrote:
> Scholars know that Sk, whoever he was, read Latin, Greek, French,
> Italian and Spanish. He very likely read Hebrew, Arabic, Dutch and
> German as well. Or as they said of HS "known in all lanauges..."

No they don't. He clearly did _not_ know Greek -- he understands Greek
culture no better than Cecil B. DeMille -- and there is zero evidence
that he knew Hebrew, Arabic, Dutch or German. There is also no evidence
that he had any fluency in Italian or Spanish, though he might have had
some reading knowledge of Italian. Even French is doubtful, outside of
H5, where it is certainly possible that he might have had some help.

David L. Webb

unread,
Oct 5, 1999, 3:00:00 AM10/5/99
to
In article <37f7e8b4....@News.localaccess.com>, john baker wrote:

> Scholars know that Sk, whoever he was, read Latin, Greek, French,
> Italian and Spanish. He very likely read Hebrew, Arabic, Dutch and
> German as well. Or as they said of HS "known in all lanauges..."

As an example, what evidence can you furnish that Shakespeare knew
Hebrew? Do you actually have any credible evidence, or are you just
making wild surmises that cannot be substantiated? I'd also be interested
in evidence of his command of Arabic and Dutch, but I'll settle for some
solid evidence of his ability to read Hebrew.

David Webb

john_baker

unread,
Oct 9, 1999, 3:00:00 AM10/9/99
to


David,

Notice how you are ducking the lanauge problem by jumping to
this claim on Hebrew. We know Sk read Latin, Green, French, Italian,
German and Spanish, gee he brought out a play on Cardenio, he nearly
quotes verbatim from Cervantes in the opening of Othello..."a round
unvarnished tale..."

But the best suggestion that Sk read Hebrew is his translation
of the 46th psalm...the one where you count in 46 words and find the
word "shakes" and then 46 from the end and find the word "spear."

The placement and the wording isn't in the Hebrew and strongly
suggests that he had a hand in translating the psalms. I believe
these were in Hebrew.

Marlowe, we know, had worked with Mary Sidney Herbert on
them back in the 1590s.

Dr. Jean Joffen, formerly with NYCC, a Hebrew schollar, has a
book long treatment on Sk's knowledge of Hebrew based on MV.

There are three cites in *Sk and the Classical Tradition*
regarding Sk's knowledge of Hebrew, two more on his knowledge of the
Hebrew religion and one on "Hebrews."

baker

But

John W. Kennedy

unread,
Oct 12, 1999, 3:00:00 AM10/12/99
to
john, baker wrote:
>
> Notice how you are ducking the lanauge problem by jumping to
> this claim on Hebrew. We know Sk read Latin, Green, French, Italian,
> German and Spanish,

No we don't, liar. We most positively know that he could _not_ read
Greek, and there is only ambiguous evidence for Italian. and little or
nothing for German and Spanish.

> But the best suggestion that Sk read Hebrew is his translation
> of the 46th psalm...the one where you count in 46 words and find the
> word "shakes" and then 46 from the end and find the word "spear."
>
> The placement and the wording isn't in the Hebrew and strongly
> suggests that he had a hand in translating the psalms. I believe
> these were in Hebrew.

This statement is completely idiotic, clearly demonstrating that you
think that all languages are merely encyphered English. I bet when
you's visitin' furrin' parts you yells to make yourself understood.

David L. Webb

unread,
Oct 13, 1999, 3:00:00 AM10/13/99
to
In article <37ff74d6...@News.localaccess.com>, john baker wrote:

> >> Scholars know that Sk, whoever he was, read Latin, Greek, French,
> >> Italian and Spanish. He very likely read Hebrew, Arabic, Dutch and
> >> German as well. Or as they said of HS "known in all lanauges..."

> > As an example, what evidence can you furnish that Shakespeare knew
> >Hebrew? Do you actually have any credible evidence, or are you just
> >making wild surmises that cannot be substantiated?

> Notice how you are ducking the lanauge problem by jumping to
> this claim on Hebrew.

I'm "ducking" nothing. As the extract above shows, you're the one
who claimed that Shakespeare knew Hebrew; I'm just asking for some
credible evidence that he did.

> We know Sk read Latin, Green, French, Italian,

> German and Spanish, gee he brought out a play on Cardenio, he nearly
> quotes verbatim from Cervantes in the opening of Othello..."a round
> unvarnished tale..."

I didn't ask about Latin, Greek, French, Italian, German, or
Spanish, although some of these are pretty weak claims; I asked *only*
about Hebrew.

> But the best suggestion that Sk read Hebrew is his translation
> of the 46th psalm...the one where you count in 46 words and find the
> word "shakes" and then 46 from the end and find the word "spear."
>
> The placement and the wording isn't in the Hebrew and strongly
> suggests that he had a hand in translating the psalms. I believe
> these were in Hebrew.

*That's* your "evidence" that Shakespeare knew Hebrew? You've
*got* to be kidding!

First, it's quite common for poets to produce translations of
works from tongues they don't know. Pasternak, for example, produced
highly regarded Russian verse translations of various Georgian poets,
despite not knowing Georgian, by the use of literal translations
furnished by scholars. There were (obviously) plenty of KJV
translators who knew Hebrew well. Even had Shakespeare signed his name
to the 46th Psalm, his knowledge of Hebrew would not be established.

Second, there is no credible evidence that Shakespeare "had a hand
in translating the psalms." The identities of all but a handful of the
scholars who produced the King James version are known, and Shakespeare
is not among them; nor would one expect him to have been -- the others
were well-educated churchmen and scholars in classical languages, which
Shakespeare was not.

Third, your "evidence" -- the numerical coincidence concerning the
46th Psalm -- would not even convince a confirmed conspiracy theorist!
Coincidences such as this are abundant, and Martin Gardner's collected
essays contain loads of coincidences of this sort that are far more
striking than this old 46-46 canard.

Fourth, Dave Kathman has remarked in this forum that some Bibles
prior the King James version exhibited the same phenomenon ("shakes" 46
words from the beginning, "spear" 46 words from the end); did
Shakespeare "have a hand" in translating those Bibles too?

> Dr. Jean Joffen, formerly with NYCC, a Hebrew schollar, has a
> book long treatment on Sk's knowledge of Hebrew based on MV.

Fine; what is the evidence?



> There are three cites in *Sk and the Classical Tradition*
> regarding Sk's knowledge of Hebrew, two more on his knowledge of the
> Hebrew religion and one on "Hebrews."

Fine; what is the evidence?

David Webb

0 new messages